Resp, CVS, GI, H+N, Derm + Neuro - Practice Qs! Flashcards

1
Q

What is the diagnostic criteria for a transudative vs. an exudative effusion?

A
  • Light’s Criteria -> if meets at least 1/3 of the criteria = exudative pleural effusion*
  • PF/serum protein ratio= >0.5
  • PF/serum LDH ratio = >0.6
  • PF LDH = >2/3 U/L
How well did you know this?
1
Not at all
2
3
4
5
Perfectly
2
Q

An 62 year old woman contacts 111 late on a Sunday evening. She tells the call handler that she has bronchiectasis and thinks she currently has an infection. She is coughing up larger than normal volumes of sputum and feels a bit breathless on exerting herself. Which course of action should the call handler take next?

a) Advise the patient to contact her own GP the next morning
b) Advise the patient to attend her nearest pharmacy to obtain a prescription for antibiotics
c) Arrange a 999 ambulance to take the patient to the Emergency Department
d) Arrange for a GP or ANP to further assess the patient by phone, video or in a consulting room
e) Arrange for a GP or ANP to further assess the patient by a home visit

A

d) Arrange for a GP or ANP to further assess the patient by phone, video or in a consulting room

How well did you know this?
1
Not at all
2
3
4
5
Perfectly
3
Q

Which of the following statements is NOT true for individuals with advanced chronic obstructive pulmonary disease?

a) Arterial PCO2 levels are elevated
b) Nitrous oxide is a safe sedative to use
c) They are said to be on hypoxic drive
d) They have decreased sensitivity to PCO2
e) They rely on their peripheral chemoreceptors for stimulating ventilation

A

b) Nitrous oxide is a safe sedative to use

How well did you know this?
1
Not at all
2
3
4
5
Perfectly
4
Q

A 55 year old obese business man complains of excessive daytime somnolence, snoring and frequent napping. He undergoes overnight oximetry which shows 10 desaturations per hour. What is his classification on the Obstructive Sleep Apnoea Severity scale?

a) Mild
b) Moderate
c) Normal - no further investigation required
d) Normal - should be referred for limited sleep study
e) Severe

A

a) Mild

AHI score:

  • None/Minimal: AHI < 5 per hour.
  • Mild: AHI ≥ 5, but < 15 per hour.
  • > 10: referral for limited sleep study, and possible CPAP
  • Moderate: AHI ≥ 15, but < 30 per hour.
  • Severe: >30
How well did you know this?
1
Not at all
2
3
4
5
Perfectly
5
Q

When should ICS be offered to asthma patients (adults + children) currently taking a SABA?

A
  • Using inhaled β2 agonists three times a week or more
  • Symptomatic 3 times/week or more or
  • Waking one night a week. In addition
  • Have had an asthma attack requiring oral corticosteroids in the last two years.
How well did you know this?
1
Not at all
2
3
4
5
Perfectly
6
Q

What are the muscular and physical forces involved in drawing air into the lungs during inspiration?

A

The external intercostal and diaphragm muscles contract and thoracic volume increases

How well did you know this?
1
Not at all
2
3
4
5
Perfectly
7
Q

A 35-year-old male asks his GP for advice about smoking cessation. He has been a smoker since the age of 15 years and smokes 10 cigarettes a day. What is his pack-year history?

A
  • 1 pack = 20 cigarettes -> 10 cigarettes = 0.5 pack
  • Smoker for 20yrs
  • 20 x 0.5 = 10 pack year
How well did you know this?
1
Not at all
2
3
4
5
Perfectly
8
Q

What type of history would suggest Chlamydia psittaci Pneumonia?

A
  • Bird handlers
How well did you know this?
1
Not at all
2
3
4
5
Perfectly
9
Q

What type of history would suggest H. influenzae Pneumonia?

A
  • COPD exacerbation
How well did you know this?
1
Not at all
2
3
4
5
Perfectly
10
Q

What type of history would suggest Legionella pneumoniae Pneumonia?

A
  • Contaminated water

- Air conditioning units

How well did you know this?
1
Not at all
2
3
4
5
Perfectly
11
Q

What type of history would suggest Pseudemonas aeruginosa Pneumonia?

A
  • often nosocomial, found in patients with severe chronic lung disease e.g. CF, bronchiectasis, end stage COPD
How well did you know this?
1
Not at all
2
3
4
5
Perfectly
12
Q

A 50 year old oil man has lobar pneumonia. Which is the most likely organism causing his condition?

A

Strep Pneumoniae

How well did you know this?
1
Not at all
2
3
4
5
Perfectly
13
Q

What does CURB-65 stand for?

A
  • used for predicting mortality in Community Acquired Pneumonia*
  • Confusion of new onset
  • Blood Urea nitrogen >7 mmol/L
  • RR of 30 breaths/minute or greater
  • BP <90 mmHg systolic or 60 mmHg or less diastolic
  • Age 65 or older

0-1: Treat as an outpatient
2: Consider a short stay in hospital or watch very closely as an outpatient
3-5: Requires hospitalisation with consideration as to whether they need to be in the intensive care unit

How well did you know this?
1
Not at all
2
3
4
5
Perfectly
14
Q

Which of the following would not be described as a physical hazard?

a) Air pollution
b) Radiation
c) Machinery
d) Noise
e) Vibration

A

c) Machinery

= a mechanical hazard

How well did you know this?
1
Not at all
2
3
4
5
Perfectly
15
Q

How do you calculate Alveolar Ventilation?

A

(Tidal Volume - Dead Space) x RR

How well did you know this?
1
Not at all
2
3
4
5
Perfectly
16
Q

Which of the following is most likely to shift the haemoglobin oxygen binding curve to the right?

a) Asthma attack
b) Hypothermia
c) Presence of foetal haemoglobin
d) Respiratory alkalosis
e) Voluntary hyperventilation

A

shift to the right = decreased affinity for oxygen

a) Asthma attack
- > increased CO2, acidosis

all the rest increases the affinity for O2

How well did you know this?
1
Not at all
2
3
4
5
Perfectly
17
Q

A 3 year old girl attends her GP with her father. Her father informs the GP that last night she was awake for several hours with a barking cough, and her breathing seemed noisy. She has also had a runny nose for a couple of days and a fever. She appears well in the surgery today, with no signs of respiratory distress. The GP makes a diagnosis of croup. What is the single best treatment option for this child?

a) A one-off dose of oral steroids
b) No treatment is required
c) A trial of a short acting beta-2-agonist
d) A 5 day course of penicillin
e) Twice daily inhaled corticosteroids

A

a) A one-off dose of oral steroids (dexamethasone)

How well did you know this?
1
Not at all
2
3
4
5
Perfectly
18
Q

At what stage of development is surfactant production first sufficient?

A

36 weeks gestation

How well did you know this?
1
Not at all
2
3
4
5
Perfectly
19
Q

At what stage of development is surfactant production first produced?

A

25 week gestation

How well did you know this?
1
Not at all
2
3
4
5
Perfectly
20
Q

At which stage of development are babies at risk of Infant RDS if they are born before this time?

A

Before 25 weeks gestation

How well did you know this?
1
Not at all
2
3
4
5
Perfectly
21
Q

What does Shunt mean?

A
  • When blood is transported through the lungs without taking part in gas exchange
  • Perfusion exceeds ventilation in L/min
  • Blood is “shunted” from the RHS heart to the left without being oxygenated (as pulmonary vessels vasoconstrict)
  • alveolar blockage/filling causes inadequate ventilation ie. due to pus, oedema, blood, tumour
How well did you know this?
1
Not at all
2
3
4
5
Perfectly
22
Q

What term describes the maximum volume of air that can be voluntarily exhaled following a maximum inspiration?

a) Expiratory Reserve Volume
b) Functional Residual Capacity
c) Residual Volume
d) Vital Capacity
e) Vital Volume

A

d) Vital Capacity

  • nb.
  • ERV = the volume of air that can be voluntarily forcibly exhaled after normal expiration
  • FRC = volume of air left in lungs after normal expiration (ERV + RV)
  • RV = volume of air left in lungs that cannot be voluntarily expired*
How well did you know this?
1
Not at all
2
3
4
5
Perfectly
23
Q

What is the approximate partial pressure of oxygen in mixed venous blood?

a) 40 mmHg (5.3kPa)
b) 46 mmHg (6.1kPa)
c) 100mmHg (13.3kPa)
d) 160 mmHg (21.3kPa)
e) 200 mmHg (26.7kPa)

A

a) 40 mmHg (5.3kPa)

How well did you know this?
1
Not at all
2
3
4
5
Perfectly
24
Q

What is the approximate partial pressure of CO2 in mixed venous blood?

A

46mm Hg

How well did you know this?
1
Not at all
2
3
4
5
Perfectly
25
Q

What is the approximate partial pressure of O2 and CO2 in systemic arterial blood?

A
  • O2 = 100mm Hg

- CO2 = 40mm Hg

How well did you know this?
1
Not at all
2
3
4
5
Perfectly
26
Q

What ventricular rate would you expect in atrial flutter?

A
  • 150bpm

-> 2:1 block = 2 atrial beats for each ventricular beat
atrial rate = 300bpm

How well did you know this?
1
Not at all
2
3
4
5
Perfectly
27
Q

What causes a “sawtooth” pattern on ECG?

A

atrial flutter

How well did you know this?
1
Not at all
2
3
4
5
Perfectly
28
Q

What is associated with atrial flutter?

A
  • HT
  • IHD
  • Cardiomyopathy
  • Thyrotoxicosis
How well did you know this?
1
Not at all
2
3
4
5
Perfectly
29
Q

What is the pathophysiology of atrial flutter?

A
  • “Re-entrant rhythm” in either atria
  • Self-perpetuating loop in an extra abnormal electrical circuit pathway
  • The signal goes round and round the atria without interruption (300bpm) -> only manages to make it to the ventricles every second lap due to long refractory period to the AV node (ventricular contraction = 150bpm)
How well did you know this?
1
Not at all
2
3
4
5
Perfectly
30
Q

What is the treatment of atrial flutter?

A
  • same as AF*
  • Rate control with beta blockers or cardioversion (if rhythm not restored with BBs)
  • Treat reversible underlying condition (e.g. hypertension or thyrotoxicosis)
  • RFA of the re-entrant rhythm (if meds not tolerated)
  • Anticoagulation based on CHA2DS2VASc score
How well did you know this?
1
Not at all
2
3
4
5
Perfectly
31
Q

What is the treatment of an acute STEMI?

A
  • Patients with STEMI presenting within 12 hours of onset should be discussed urgently with local cardiac centre for either:*
  • Primary PCI (if available within 2 hours of presentation)
  • Thrombolysis (if PCI not available within 2 hours)
How well did you know this?
1
Not at all
2
3
4
5
Perfectly
32
Q

An 80 year old woman with chest pain has an elevated troponin on blood testing. Which is a non-cardiac cause of elevated troponin?

a) Chronic kidney disease
b) Diverticulosis
c) Liver failure
d) Myocarditis
e) Pituitary adenoma

A

a) CKD
* nb. Myocarditis IS a cardiac cause!!*
* whilst cardiac ischaemia and cardiac muscle damage can cause elevated troponin, it can be elevated by other factors, including CKD. The reason is not clear - it may be that uraemia causes damage to skeletal muscle and release of troponin*

How well did you know this?
1
Not at all
2
3
4
5
Perfectly
33
Q

What are some causes of elevated Troponin which are not due to acute CAD?

A
  • Myocarditis
  • Pulmonary Embolism
  • Acute HF
  • Sepsis and septic shock
  • Hypovolemia
  • Renal failure (CKD)
  • AF
  • Cardiac contusion
  • > (bruising, caused by blunt damage to the heart muscle!)
How well did you know this?
1
Not at all
2
3
4
5
Perfectly
34
Q

A 70 year old man presents with an ulcer of his left leg. His sleep is being interrupted by severe leg pain. Which is the most likely cause of his ulcer?

a) Chronic venous insufficiency
b) Diabetic neuropathy
c) Critical limb ischaemia
d) Post-phlebitic syndrome
e) Squamous carcinoma

A

c) Critical limb ischaemia

  • pain results from inadequate arterial supply
  • worsened by elevating the leg -> pt. may complain of worsening pain in bed*
  • nb. venous and diabetic ulcers DO NOT tend to cause severe pain*
  • post-phlebitic syndrome is a symptom of chronic venous insufficiency 2ndary to DVT*
How well did you know this?
1
Not at all
2
3
4
5
Perfectly
35
Q

What is the most common cause of Cor Pulmonale?

A
  • COPD
How well did you know this?
1
Not at all
2
3
4
5
Perfectly
36
Q

What are the signs of Cor Pulmonale?

A
  • SOB
  • Hypoxia
  • raised JVP
  • Hepatomegaly
How well did you know this?
1
Not at all
2
3
4
5
Perfectly
37
Q

What is the definition of 2:1 block?

A
  • QRS after every 2nd p wave

- p waves remain normal

How well did you know this?
1
Not at all
2
3
4
5
Perfectly
38
Q

What is the definition of 1st degree heart block?

A
  • delayed AV conduction

- PR interval >0.2s (5 small/1 big square)

How well did you know this?
1
Not at all
2
3
4
5
Perfectly
39
Q

What is the definition of Wenckebach’s phenomenon (Mobitz Type 1)

A
  • increasing PR interval until p wave no longer conducts to the ventricles
  • > absent QRS complex after a p wave
How well did you know this?
1
Not at all
2
3
4
5
Perfectly
40
Q

What is the definition of Mobitz Type 2?

A
  • intermittent failure or interruption of AV conduction (random missing QRS complexes)
  • PR interval remains normal
  • risk of Asystole
How well did you know this?
1
Not at all
2
3
4
5
Perfectly
41
Q

What is the definition of 3rd degree heart block?

A
  • Complete heart block
  • no relatonship between p waves and QRS complexes
  • risk of asystole
How well did you know this?
1
Not at all
2
3
4
5
Perfectly
42
Q

What is the medical intervention of HT for a <55, non-black pt?

A
  • Initially: A or B
  • Stage 2: A+C (use D if C not tolerated)
  • Stage 3: A + C + D
  • Stage 4: A + C + D + D
    • A = ACE inhibitor (e.g. Ramipril 1.25mg up to 10mg once daily)
  • B = Beta blocker (e.g. bisoprolol 5mg up to 20mg once daily)
  • C = Calcium channel blocker (e.g. amlodipine 5mg up to 10mg once daily)
  • D = Thiazide-Like Diuretic (e.g. indapamide 2.5mg once daily)
  • ARB = Angiotensin Receptor 2 Antagonist (e.g. candesartan 8mg to up 32mg once daily)*
How well did you know this?
1
Not at all
2
3
4
5
Perfectly
43
Q

What is the medical intervention of HT for an >55 pt?

A
  • Initially: C
  • Stage 2: A+C
  • Stage 3: A + C + D (use D if C not tolerated)
  • Stage 4: A + C + D + D
    • A = ACE inhibitor (e.g. Ramipril 1.25mg up to 10mg once daily)
  • B = Beta blocker (e.g. bisoprolol 5mg up to 20mg once daily)
  • C = Calcium channel blocker (e.g. amlodipine 5mg up to 10mg once daily)
  • D = Thiazide-Like Diuretic (e.g. indapamide 2.5mg once daily)
  • ARB = Angiotensin Receptor 2 Antagonist (e.g. candesartan 8mg to up 32mg once daily)*
How well did you know this?
1
Not at all
2
3
4
5
Perfectly
44
Q

What is the medical intervention of HT for a black pt?

A
  • Initially: C
  • Stage 2: ARB +C
  • Stage 3: A + C + D (use D if C not tolerated)
  • Stage 4: A + C + D + D
    • A = ACE inhibitor (e.g. Ramipril 1.25mg up to 10mg once daily)
  • B = Beta blocker (e.g. bisoprolol 5mg up to 20mg once daily)
  • C = Calcium channel blocker (e.g. amlodipine 5mg up to 10mg once daily)
  • D = Thiazide-Like Diuretic (e.g. indapamide 2.5mg once daily)
  • ARB = Angiotensin Receptor 2 Antagonist (e.g. candesartan 8mg to up 32mg once daily)*
How well did you know this?
1
Not at all
2
3
4
5
Perfectly
45
Q

What is first line treatment of acute heart failure?

A
  • IV Furosemide (diuretic)
How well did you know this?
1
Not at all
2
3
4
5
Perfectly
46
Q

What is first line treatment of chronic heart failure?

A
  • ACEI and BB (ie. bisoprolol)
  • ARB if ACEI not tolerated
  • > (hydrazaline hydrochloride can be used if black and/or intolerant of ACEIs and ARBs)
How well did you know this?
1
Not at all
2
3
4
5
Perfectly
47
Q

What is initial add-on therapy of chronic heart failure?

A
  • Aldosterone (MR) antagonist (ie. spironolactone)

- > (if no contra-indications ie. due to hyperkalaemia or renal impairment)

How well did you know this?
1
Not at all
2
3
4
5
Perfectly
48
Q

What is the recommended as add-on therapy in worsening or severe chronic heart failure, despite optimal treatment?

A

Digoxin!

-> (nb. needs to be in sinus rhythm)

How well did you know this?
1
Not at all
2
3
4
5
Perfectly
49
Q

Which medical therapies provide symptomatic relief in chronic HF?

A
  • Loop diuretics
  • > ie. Furosemide
  • Digoxin
  • > severe (HF refractive to other medical therapies)
How well did you know this?
1
Not at all
2
3
4
5
Perfectly
50
Q

What is the normal duration of a QRS complex?

a) > 0.8 seconds
b) < 1.5 seconds
c) < 0.12 seconds
d) > 0.1 seconds
e) < 0.8 seconds

A

c) < 0.12 seconds

<3 small squares

How well did you know this?
1
Not at all
2
3
4
5
Perfectly
51
Q

What is the normal duration of a PR interval?

A

0.12-0.20 seconds

3-5 small squares

How well did you know this?
1
Not at all
2
3
4
5
Perfectly
52
Q

A 74 year old woman with uncontrolled diabetes already on amlodipine and ramipril presents with uncontrolled hypertension. Which of the following medications is most appropriate to start next?

a) Amlodipine
b) Ramipril
c) Candesartan
d) Indapamide
e) Atorvastatin

A

d) Indapamide

How well did you know this?
1
Not at all
2
3
4
5
Perfectly
53
Q

You are asked to review a patient with a heart rate of 160 bpm.They are otherwise haemodynamically stable and you decide to treat. The ECG reveals a regular, broad based tachycardia with QRS complexes around 0.25 seconds. What would be an appropriate initial intervention in this scenario?

a) IV Adenosine
b) IV Atropine
c) IV Metoprolol
d) IV Adrenalin
e) IV Amiodarone

A

a) IV Adenosine

How well did you know this?
1
Not at all
2
3
4
5
Perfectly
54
Q

A 26 year old woman has a hot, swollen right leg following a flight from Australia. Ultrasound confirms deep vein thrombosis (DVT) in the right lower limb. Which is the most appropriate initial treatment?

a) Aspirin
b) Clopidogrel
c) LMWH
d) Thrombolysis
e) Warfarin

A

c) LMWH
* Thrombolysis is used for a life-threatening massive PE*
* dalteparin and warfarin are superceded by DOACs*
* aspirin and clopidogrel = antiplatelets (no place in the management of VTE!*

How well did you know this?
1
Not at all
2
3
4
5
Perfectly
55
Q

A child is born with coarctation of the aorta, short stature, neck webbing and gonadal dysgenesis. Which is the most likely diagnosis?

A

Turner syndrome

How well did you know this?
1
Not at all
2
3
4
5
Perfectly
56
Q

What is Noonan syndrome associated with?

A

Pumonary stenosis and septal defects

How well did you know this?
1
Not at all
2
3
4
5
Perfectly
57
Q

What is Shprintzen syndrome associated with?

A

Tetralogy of Fallot

How well did you know this?
1
Not at all
2
3
4
5
Perfectly
58
Q

What is Williams syndrome associated with?

A

Supravalvular Aortic stenosis

How well did you know this?
1
Not at all
2
3
4
5
Perfectly
59
Q

You are counselling a patient with severe mitral regurgitation on the pros and cons of replacement heart valves. What is the most significant issue with mechanical heart valves when compared to bioprosthetic valves?

a) Arrhythmias
b) Limited lifespan
c) Early failure after surgery
d) Thrombus formation
e) Infective endocarditis

A

d) Thrombus formation

How well did you know this?
1
Not at all
2
3
4
5
Perfectly
60
Q

What is the anti-coagulation of choice for mechanical heart valves?

What target INR is required for it?

A
  • Warfarin

- 2.5-3.5

How well did you know this?
1
Not at all
2
3
4
5
Perfectly
61
Q

A 43 year old black diabetic woman is confirmed to have hypertension. Which of the following medications is most appropriate to start first-line?

a) Amlodipine
b) Ramipril
c) Candesartan
d) Indapamide
e) Atorvastatin

A

c) Candesartan

<55, and has diabetes BUT is also black so no ACEIs, so use ARB instead!

How well did you know this?
1
Not at all
2
3
4
5
Perfectly
62
Q

What blood test is most important in a patient who is starting and ACE inhibitor?

a) LFT
b) G6PD
c) Antiphospholipid antibodies
d) U+E
e) FBC

A

d) U+E

kidney profile

How well did you know this?
1
Not at all
2
3
4
5
Perfectly
63
Q

A child is born with atrioventricular septal defects. Which is the most likely diagnosis?

a) Down syndrome
b) Noonan syndrome
c) Shprintzen syndrome
d) Turner syndrome
e) Williams syndrome

A

a) Down syndrome

How well did you know this?
1
Not at all
2
3
4
5
Perfectly
64
Q

What sort of pain does arterial thromboembolus produce? (CLI)

A

Sudden severe pain, not related to exertion (in lower limb)

How well did you know this?
1
Not at all
2
3
4
5
Perfectly
65
Q

What sort of pain does DVT produce?

A

Constant pain (in lower limb)

How well did you know this?
1
Not at all
2
3
4
5
Perfectly
66
Q

What sort of pain do Varicose veins produce?

A

Leg pain associated with long periods of standing, rather than related to exertion

How well did you know this?
1
Not at all
2
3
4
5
Perfectly
67
Q

What sort of pain does PVD produce?

A

Intermittent claudication (lower limbs)

How well did you know this?
1
Not at all
2
3
4
5
Perfectly
68
Q

A 60 year old man is prescribed amlodipine (calcium channel blocker) after ambulatory blood pressure monitoring revealed stage 2 hypertension. How does amlodipine reduce blood pressure?

a) By increasing cardiac output
b) By increasing diuresis
c) By reducing heart rate
d) By reducing sympathetic drive
e) By reducing vascular resistance

A

e) By reducing vascular resistance

How well did you know this?
1
Not at all
2
3
4
5
Perfectly
69
Q

You diagnose essential hypertension in a 45 year old black female patient who has ambulatory blood pressure monitoring demonstrating an average blood pressure of 156/89. She is otherwise fit and well and has no known allergies. After discussing lifestyle interventions, you agree with the patient that medication should be commenced. What would be the most appropriate first line medication from the list below?

a) Bisoprolol
b) Ramipril
c) Aspirin
d) Amlodipine
e) Candesartan

A

d) Amlodipine

Black

How well did you know this?
1
Not at all
2
3
4
5
Perfectly
70
Q

What event causes the first heart sound?

A

Closure of the atrioventricular valves

How well did you know this?
1
Not at all
2
3
4
5
Perfectly
71
Q

What event causes the second heart sound?

A

Closure of the semilunar valves

How well did you know this?
1
Not at all
2
3
4
5
Perfectly
72
Q

In the heart, the fast depolarising phase of the cardiac action potential is caused by the influx of which ions?

A

influx of Na+

How well did you know this?
1
Not at all
2
3
4
5
Perfectly
73
Q

In the heart, the RMP of the cardiac myocyte is caused by the efflux of which ions?

A

efflux of K+

-> “leaky” K+ channels

How well did you know this?
1
Not at all
2
3
4
5
Perfectly
74
Q

In the heart, the plateau phase of the cardiac action potential is caused by which channels?

A
  • Opening of L-type voltage-gated Ca2+ channels and closure of some leaky K+ channels
How well did you know this?
1
Not at all
2
3
4
5
Perfectly
75
Q

In the heart, the repolarisation phase of the cardiac action potential is caused by which channels?

A
  • Closing of L-type voltage-gated Ca2+ channels and opening of some leaky K+ channels
How well did you know this?
1
Not at all
2
3
4
5
Perfectly
76
Q

Which of the following is an example of a social policy the government has put in place to improve health?

a) Banning advertisement of unhealthy food
b) Cycle to work schemes
c) Minimum alcohol pricing
d) Free school meals for children in holidays
e) Winter fuel allowance

A

a) Banning advertisement of unhealthy food

    • Cycle to work schemes, winter fuel allowance, free school meals are examples of subsidies.
  • Minimum alcohol pricing is an example of a tax.
  • Banning advertisement of unhealthy food is the only example of social policy*
How well did you know this?
1
Not at all
2
3
4
5
Perfectly
77
Q

A 58-year-old builder consults his GP. He has a three-month history of chest discomfort when exerting himself at work. He also reports slight breathlessness when climbing stairs. If he stops and rests both the chest discomfort and breathlessness settle after a couple of minutes. He has never felt breathless or had chest discomfort at rest. You think it is likely that he has stable angina. Which two investigations would be the most appropriate first line investigations to organise in primary care?

a) ECG and CXR
b) ECG and blood tests
c) CXR and blood tests
d) 24 hour holter monitor and blood tests
e) ECHO and blood tests

A

b) ECG and blood tests

-> lipid profile for full CV risk

How well did you know this?
1
Not at all
2
3
4
5
Perfectly
78
Q

What is the first line investigation for a new heart murmur?

A

ECHO

How well did you know this?
1
Not at all
2
3
4
5
Perfectly
79
Q

What are the investigations for suspected HF?

A
  • Blood test: B-Natriuretic peptide (BNP) and ECG

- If either is abnormal: then ECHO
if both normal: less likely to be HF

How well did you know this?
1
Not at all
2
3
4
5
Perfectly
80
Q

What is the most common side-effect of CCBs?

A

Ankle Oedema

How well did you know this?
1
Not at all
2
3
4
5
Perfectly
81
Q

What is the treatment of stable SVT?

A
  • Valsalva maneouvre

- if that doesn’t work: IV Adenosine

How well did you know this?
1
Not at all
2
3
4
5
Perfectly
82
Q

What blood investigation will be most helpful in diagnosing chronic heart failure?

A

BNP

How well did you know this?
1
Not at all
2
3
4
5
Perfectly
83
Q

What is the classification of Heart Failure?

A
  • NYHA score*
  • Type I: No limitation of physical activity. Ordinary physical activity does not cause undue fatigue, palpitation, dyspnoea (shortness of breath).
  • Type II: Slight limitation of physical activity. Comfortable at rest. Ordinary physical activity results in fatigue, palpitation, dyspnea (shortness of breath).
  • Type III: Marked limitation of physical activity. Comfortable at rest. Less than ordinary activity causes fatigue, palpitation, or dyspnea.
  • Type IV: Unable to carry on any physical activity without discomfort. Symptoms of heart failure at rest. If any physical activity is undertaken, discomfort increases.
How well did you know this?
1
Not at all
2
3
4
5
Perfectly
84
Q

Other than AF, what can cause an irregularly irregular pulse?

A
  • Ventricular ectopics
How well did you know this?
1
Not at all
2
3
4
5
Perfectly
85
Q

A 64 year old presents complaining of shortness of breath, worse on exertion and when lying flat at night. He is known to have ischaemic heart disease and is on medication for angina. He has had two previous NSTEMIs. On auscultation you hear a grade 3, pan-systolic murmur loudest at the apex. What is the most likely cause for his murmur?

a) Aortic stenosis
b) Mitral stenosis
c) Tricuspid regurgitation
d) Mitral regurgitation
e) Aortic regurgitation

A

d) Mitral regurgitation

MR. ASS

How well did you know this?
1
Not at all
2
3
4
5
Perfectly
86
Q

What is Mitral Regurgitation associated with?

A
  • Pan-systolic, high pitched “whistling” murmur (high velocity blood through leaky valve)
  • Radiates to left axilla
  • Results in congestive cardiac failure (because leaking valve causes reduced ejection fraction and back-pressure)
  • Associated with third heart sound (causes heart failure)
How well did you know this?
1
Not at all
2
3
4
5
Perfectly
87
Q

What are the causes of Mitral Regurgitation?

A
  • Idiopathic weakening of valve with age
  • Ischaemic heart disease
  • Infective Endocarditis
  • Rheumatic Heart Disease
  • Connective tissue disorders (e.g. Ehlers Danlos or Marfan’s)
How well did you know this?
1
Not at all
2
3
4
5
Perfectly
88
Q

A 68 year old man presents with exertional angina. Which is the most appropriate treatment to improve symptoms?

a) Angiotensin-converting-enzyme (ACE) inhibitor
b) Aspirin 75mg/day
c) Beta blocker
d) Low dose warfarin
e) Simvastatin

A

c) Beta blocker
* Beta blockers are negatively inotropic, and so are the only option which reduce the workload of the heart. The others help reduce risk factors, but beta blockers, by reducing workload, reduce oxygen requirements/less ischaemia/ less pain. There is no evidence for warfarin use being beneficial in ACS*

How well did you know this?
1
Not at all
2
3
4
5
Perfectly
89
Q

How to differentiate between angina, unstable angina, N-STEMI and STEMI?

A
  • Angina = central crushing chest pain; precipitated by exercise, cold weather, stress; radiates to shoulder; relieved by rest, GTN spray, or both
  • Unstable Angina = same as above, but not relieved by rest or GTN spray, normal bloods and ECG
  • N-STEMI = same as above, bloods show elevated troponins, ECG doesn’t show ST elevation
  • STEMI = same as unstable angina, bloods show elevated troponins, ECG does show ST elevation
How well did you know this?
1
Not at all
2
3
4
5
Perfectly
90
Q

In the cardiac cycle, which of the following corresponds to the period between closure of the aortic valve and opening of the mitral valve?

a) Active filling phase
b) Diastole
c) Isometric contraction phase
d) Isometric relaxation phase
e) Systole

A

d) Isometric relaxation phase
* The valves in the heart are purely passive and open or close according to the pressure each side of them. The aortic valve lies at the exit from the left ventricle, separating the left ventricle from the aorta. The mitral valve lies at the entrance to the left ventricle, separating the left atrium from the left ventricle.

Once you have that diagram in your mind, you should realise that the aortic valve closes at the very start of the ventricular relaxation phase, when ventricular pressure falls below aortic pressure, and that the mitral valve opens later in the relaxation phase, when left ventricular pressure is below left atrial pressure. In the interval between the two events, the valves at the entrance and exit from the ventricle is closed and so the volume cannot change. This therefore describes the isometric relaxation phase.*

How well did you know this?
1
Not at all
2
3
4
5
Perfectly
91
Q

What is the 1st line treatment of AF?

A
  • Atenolol
How well did you know this?
1
Not at all
2
3
4
5
Perfectly
92
Q

What medications are appropriate for rate-control in AF?

A
  • BBs
  • CCBs
  • Digoxin (if other drugs unsuitable)
How well did you know this?
1
Not at all
2
3
4
5
Perfectly
93
Q

What medications are appropriate for rhythm control in AF?

A
  • Flecainide (class 1c anti-arrhythmic)

- Sotalol (BB)

How well did you know this?
1
Not at all
2
3
4
5
Perfectly
94
Q

A 40 year old man is found to have high blood pressure. You believe he may suffer from “white coat hypertension”. Which investigation would confirm this?

a) 24 hour ambulatory blood pressure recording
b) 24 hour urinary catecholamines
c) Echocardiography
d) Renal arteriography
e) Renal ultrasound

A

24 hr ambulatory BP recording

Confirmation of hypertension is needed first and this should be done with 24 hour ambulatory monitoring. If hypertension is confirmed then it would be considered whether this is primary (idiopathic) or secondary to another condition in which case the other investigations could be considered.

How well did you know this?
1
Not at all
2
3
4
5
Perfectly
95
Q

Which symptoms suggest gastric and duodenal ulcers?

Which symptoms would suggest gastric vs. duodenal ulcers?

A

both ulcers:

  • dark, tar-like stools and worsening epigastric pain
  • vomiting of blood

differences:

  • GU: epigastric pain is EXACERBATED by eating
  • DU: epigastric pain is IMPROVED by eating
How well did you know this?
1
Not at all
2
3
4
5
Perfectly
96
Q

How does the CFTR channel protein work?

A
  • Responsible for the movement of Cl- and H2O out of the apical membrane of the Crypt cells

How it works:

  • Na+-K+-Cl-co-transporter moves Cl- into the cell
  • Build-up of negative gradient inside the cell
  • Movement of Cl- through the apical membrane via CFTR
  • Regulation of CFTR via Adenylyl Cyclase
  • The gradient created by Cl- allows H2O to follow osmotically
How well did you know this?
1
Not at all
2
3
4
5
Perfectly
97
Q

What conditions is PSC associated with?

A
  • UC

- Cholangiocarcinoma

How well did you know this?
1
Not at all
2
3
4
5
Perfectly
98
Q

What is the gold-standard investigation for PSC?

A

MRCP

How well did you know this?
1
Not at all
2
3
4
5
Perfectly
99
Q

Which hormones are responsible for delayed gastric emptying?

A
  • CCK (!!!)

- Secretin

How well did you know this?
1
Not at all
2
3
4
5
Perfectly
100
Q

What is the classical presentation of Primary Biliary Cholangitis?

a) 30-40 year old female with autoimmune diseases
b) 60-70 year old male with a history of Ulcerative Colitis
c) 10-20 year old female with autoimmune diseases
d) 30-40 year old male with a history of Ulcerative Colitis
e) Neonates with autoimmune diseases

A

a) 30-40 year old female with autoimmune diseases

How well did you know this?
1
Not at all
2
3
4
5
Perfectly
101
Q

What is the classical presentation of Primary Sclerosing Cholangitis?

a) 30-40 year old female with autoimmune diseases
b) 60-70 year old male with a history of Ulcerative Colitis
c) 10-20 year old female with autoimmune diseases
d) 30-40 year old male with a history of Ulcerative Colitis
e) Neonates with autoimmune diseases

A

d) 30-40 year old male with a history of Ulcerative Colitis

How well did you know this?
1
Not at all
2
3
4
5
Perfectly
102
Q

A 55 year old man is seen by his GP due to a longstanding history of heartburn and reflux. He has no other medical problems. His GP organises a gastroscopy, which reveals high-grade dysplasia. What is the most appropriate next step?

a) Endoscopic ablation/resection
b) High dose omeprazole and annual endoscopic surveillance
c) High dose steroids.
d) High dose omeprazole and six-monthly endoscopic surveillance.
e) Five-yearly endoscopic surveillance.

A

a) Endoscopic ablation/resection
* according to the BSG: in pts with Barrett’s Oesophagus showing high-grade dysplasia -> endoscopic resection is the management of choice*

How well did you know this?
1
Not at all
2
3
4
5
Perfectly
103
Q

Which one is TRUE about salivary secretion?

a) Vagal stimulation results in stimulation of a profuse watery salivary secretion.
b) The facial and glossopharyngeal nerves stimulate secretion of watery saliva
c) The sympathetic system results in stimulation of a profuse watery salivary secretion
d) Vagal stimulation results in stimulation of a thick salivary secretion
e) The parasympathetic system results in stimulation of a thick salivary secretion

A

b) The facial and glossopharyngeal nerves stimulate secretion of watery saliva
* the vagus nerve does not innervate the head and neck!!*

How well did you know this?
1
Not at all
2
3
4
5
Perfectly
104
Q

What is source and action of Parasympathetic and Sympathetic control of Salivary secretion?

A

Parasympathetic:

  • Source: Facial (VII) and Glossopharyngeal (IX) nerve
  • Action: profuse watery secretion

Sympathetic:

  • Source: mucus via a-1 adrenoreceptors + amylase via b-2 adrenoreceptors
  • Action: small volume, viscous salivary secretion
How well did you know this?
1
Not at all
2
3
4
5
Perfectly
105
Q

Which part of the stomach produces Gastrin?

A

Gastric Antrum

G cells

How well did you know this?
1
Not at all
2
3
4
5
Perfectly
106
Q

Which part of the stomach produces Pepsinogen?

A

Gastric Body

chief cells

How well did you know this?
1
Not at all
2
3
4
5
Perfectly
107
Q

What does ALARMS stand for?

A
  • Anaemia
  • Loss of weight
  • Anorexia (loss of appetite)
  • Recent onset of progressive symptoms
  • Masses & Malaena (Black colored stools)/Haematemesis (Blood vomit)
  • Swallowing difficulty (Dysphagia)
How well did you know this?
1
Not at all
2
3
4
5
Perfectly
108
Q

What is the classical presentation of a patient with Crohn’s disease?

a) Bloody diarrhoea with mucus and weight loss
b) Epigastric pain and dysphagia
c) Constipation and haematemesis
d) Non-bloody diarrhoea with weight loss
e) Heartburn

A

d) Non-bloody diarrhoea with weight loss

* NOT bloody diarrhoea w mucus!! = UC!!*

How well did you know this?
1
Not at all
2
3
4
5
Perfectly
109
Q

What is the current NHS screening programme for FIT testing?

A
  • FIT test every 2yrs for men and women aged 60-75y/o

- If positive -> referral for colonoscopy

How well did you know this?
1
Not at all
2
3
4
5
Perfectly
110
Q

What is the characteristic appearance of Achalasia on Barium Swallow?

A

Bird’s beak appearance

How well did you know this?
1
Not at all
2
3
4
5
Perfectly
111
Q

What is ALT:AST >1 associated with?

A

NAFLD

How well did you know this?
1
Not at all
2
3
4
5
Perfectly
112
Q

What is AST:ALT >2 associated with?

A

ALD

How well did you know this?
1
Not at all
2
3
4
5
Perfectly
113
Q

What is a cholestatic picture with only raised ALP associated with?

A

PSC and PBC

How well did you know this?
1
Not at all
2
3
4
5
Perfectly
114
Q

What is…

a) Lactose
b) Sucrose
c) Maltose

comprised of?

A

a) Lactose = galactose + glucose
b) Sucrose = fructose + glucose
c) glucose + glucose

How well did you know this?
1
Not at all
2
3
4
5
Perfectly
115
Q

What is the first-line treatment for a haemodynamically stable pt. with bleeding oesophageal varices?

A

Terlipressin and broad-spectrum abx

if haemodynamically unstable and not about to receive endoscopy (band ligation), then a Sengsten-Blakemore tube would be appropriate

How well did you know this?
1
Not at all
2
3
4
5
Perfectly
116
Q

What is the function of GLUT-5?

A

Transporter of Fructose INTO the intestinal epithelial cell

Five for Fructose

How well did you know this?
1
Not at all
2
3
4
5
Perfectly
117
Q

A 43 year old alcoholic is admitted with right upper quadrant pain. Eight hours after his admission you are called to see him because he has become anxious and confused. He has started having visual hallucinations and is quite distressed. What medication is the most appropriate to help with the symptoms this patient is experiencing?

a) Chlordiazepoxide
b) Lorazepam
c) Midazolam
d) Haloperidol
e) Clonazepam

A

a) Chlordiazepoxide
- > benzodiazepine

(bc this pt. has alcohol withdrawal symptoms!*

How well did you know this?
1
Not at all
2
3
4
5
Perfectly
118
Q

What is the classical presentation of Haemorrhoids?

A

Fresh blood in the stool, or on wiping, with NO pain during defecation

How well did you know this?
1
Not at all
2
3
4
5
Perfectly
119
Q

What is the classical presentation of Diverticulitis?

A
  • Localised ABDOMINAL PAIN due to inflammation of the diverticula, along with FEVER, nausea, decreased appetite.
  • Painless bleeding from rectum
  • Mucus/blood in stool
How well did you know this?
1
Not at all
2
3
4
5
Perfectly
120
Q

What is the classical presentation of Anal Fissures?

A

Fresh blood in the stool, or on wiping, with SEVERE pain during defecation

How well did you know this?
1
Not at all
2
3
4
5
Perfectly
121
Q

What is the classical presentation of UC?

A
  • Bloody diarrhoea with mucus

- Weight loss

How well did you know this?
1
Not at all
2
3
4
5
Perfectly
122
Q

What is the classical presentation of Colon cancer?

A
  • Progressive symptoms!!

ie. rectal bleeding which worsens and is accompanied by pain
+ weight loss, fatigue

How well did you know this?
1
Not at all
2
3
4
5
Perfectly
123
Q

Which of the following is FALSE regarding vitamins?

a) Fat-soluble vitamins are processed as fat and stored in micelles
b) Water-soluble vitamins are absorbed by active transport
c) Fat-soluble vitamins include vitamins: A, D, E, K
d) Vitamin B12 binds to intrinsic factor and is absorbed in the distal ileum
e) Water-soluble vitamins include vitamin B and vitamin C

A

b) Water-soluble vitamins are absorbed by active transport
* Water-soluble vitamins, like B group, C and folic acid, are absorbed by either passive diffusion or carrier-mediated transport.

Fat soluble vitamins, like A ‒ D ‒ E ‒ K, are processed as fat and stored in micelles then chylomicrons, along with TAGs, going into lacteals.

Vitamin B12 binds to intrinsic factor in the stomach to form a complex which is absorbed via specific transport mechanism in distal ileum.*

How well did you know this?
1
Not at all
2
3
4
5
Perfectly
124
Q

A 25 year old man presents with difficulty swallowing of both food and drinks. He complains of feeling chest discomfort and occasional regurgitation. He is otherwise well, and has not complained of any weight loss. A barium swallow was done to investigate, which shows the following bird beak appearance. What additional investigation is most likely to confirm the diagnosis?

a) 24-hr pH study
b) Chest x-ray
c) Oesophageal manometry
d) Endoscopy
e) CT scan

A

c) Oesophageal manometry

* Achalasia!!*

How well did you know this?
1
Not at all
2
3
4
5
Perfectly
125
Q

What are the clinical features of Achalasia?

A
  • Dysphagia (both solids and liquids)
  • Regurgitation
  • Cough
  • Chest pain
  • due to failure of the LOS to relax!!*
How well did you know this?
1
Not at all
2
3
4
5
Perfectly
126
Q

What is Faecal calprotectin used for?

A
  • Used to differentiate between IBS and IBD!!

- Has a high negative predictive value for ruling out IBD

How well did you know this?
1
Not at all
2
3
4
5
Perfectly
127
Q

A 25 year old man presents with fever, bloody diarrhoea and cramping for several weeks that does not resolve with antibiotic therapy. Colonoscopy is diagnostic of Ulcerative colitis. Which complication is associated with this condition?

a) Primary Sclerosing Cholangitis
b) Colorectal Cancer
c) Transmural inflammation
d) Skip lesions
e) Oral ulcers

A

a) Primary Sclerosing Cholangitis
* Colorectal cancer is not, but rather cholangiocarcinoma in (10-20% of cases of UC)

The other three signs are associated with Crohn’s*

How well did you know this?
1
Not at all
2
3
4
5
Perfectly
128
Q

One of the following molecules is produced in the duodenum and it does not directly regulate the Hydrogen/Potassium ATPase

a) Gastrin
b) Secretin
c) Histamine
d) Acetylcholine
e) Prostaglandin

A

b) Secretin
* Secretin is produced by S cells in the duodenum which can affect the Hydrogen/Potassium pump in the stomach indirectly by inhibiting gastrin secretion leading to decreased gastric acid secretion

The rest of the molecules directly act on the Hydrogen/Potassium pump, all of them are stimulatory except Prostaglandin which is inhibitory.

Gastrin and Acetylcholine bind to a receptor that increase Ca, which acts on Protein Kinase C, increasing the activity of ATPase

Histamine binds to a receptor that initiates Gs-coupled reaction, stimulating adenlyl cyclase conversion of ATP to cAMP, cAMP acts on Protein Kinase A, increasing the activity of ATPase.

Prostaglandin binds to a receptor that initiates Gi-coupled reaction, inhibitng adenlyl cyclase conversion of ATP to cAMP, decreasing the activity of ATPase.*

How well did you know this?
1
Not at all
2
3
4
5
Perfectly
129
Q

What test is diagnostic of IBD?

A
  • Colonoscopy w biopsy
How well did you know this?
1
Not at all
2
3
4
5
Perfectly
130
Q

If a barium swallow shows a “corkscrew” appearance, what is the most likely diagnosis?

A
  • Diffuse Oesophageal Spasm
How well did you know this?
1
Not at all
2
3
4
5
Perfectly
131
Q

What is the difference between Crohn’s and Coeliac disease?

A

Both:
- episodes of diarrhoea with mucus and weight loss, tiredness and ulcers

Differentiating feature:
- gluten-free diet improves symptoms in Coeliac!!

How well did you know this?
1
Not at all
2
3
4
5
Perfectly
132
Q

What is the difference between squamous cell carcinoma and adenocarcinoma of the oesophagus?

A
  • SCC = malignant tumour of the MIDDLE 1/3 oesophagus

- Adenocarcinoma = malignant tumour of the DISTAL oeophagus (2ndary to Barrett’s Oesophagus)

How well did you know this?
1
Not at all
2
3
4
5
Perfectly
133
Q

Which pumps are responsible for gastric acid secretion?

A

Hydrogen/potassium adenosine triphosphatase pumps

134
Q

What is the diagnostic test for Coeliac disease?

What are the characteristic findings of the disease?

A
  • Endoscopy w Duodenal biopsy!!

- Crypt hypertrophy and villous atrophy!!

135
Q

Which of the following is FALSE regarding the secretion of HCl by Parietal cells?

a) Chloride moves into the Parietal cell in exchange for Bicarbonate
b) Carbonic Anhydrase plays a role in the secretion of HCl
c) Hydrogen moves into the
d) Stomach lumen in exchange for Potassium
pH of the Stomach can go down to <2
e) Chloride moves into the bloodstream in exchange for Bicarbonate

A

e) Chloride moves into the bloodstream in exchange for Bicarbonate

(FALSE: bc Cl- acc moves into the PARIETAL CELL in exchange for bicarbonate -> HCO3- into the bloodstream (post-prandial alkalisation))

136
Q

Which ion is the majority of amino acid absorption dependant on?

A

Na+

secondary active transport

137
Q

What type of cancer is PBC associated with?

A

HCC

nb. PSC is associated with cholangiocarcinoma

138
Q

Which of the following is FALSE regarding the Gastric phase of gastric acid secretion?

a) The gastric phase is a self-limiting phase.
b) The distension of the stomach makes up the gastric phase
c) Vagal reflexes increase acetylcholine secretion
d) The presence of peptides in the stomach lumen increases gastrin secretion
e) Gastrin and Acetylcholine act on enterochromaffin-like cells to inhibit Histamine

A

e) Gastrin and Acetylcholine act on enterochromaffin-like cells to inhibit Histamine

Gastric Phase

→ Distension of stomach:

[1] vagal/enteric reflexes = release of ACh in Parietal cells

[2] peptides in lumen = ↑ G cells = ↑ Gastrin in Parietal cells

[3] Gastrin + ACh = ↑ ECL cells = produce Histamine in Parietal cells

[4] ↓ pH (i.e. HCl) = ↓ Gastrin (self-limiting)

139
Q

A 70 year old woman presents with jaundice. She has been complaining of right abdominal pain and altered bowel habit for several months. On examination she has a hard craggy mass in her right iliac fossa and hepatomegaly. An abdominal ultrasound is performed. Which one of the following is the most likely finding in the liver at ultrasound?

a) Cholecystitis
b) Gallstones impacted in the common bile duct
c) Liver abscess
d) Macronodular cirrhosis
e) Multiple liver metastases

A

e) Multiple liver metastases (!!!)

nb. hard craggy mass!

140
Q

What are anti-TTG antibodies associated with?

A

Coeliac disease

141
Q

What type of anaemia does Type A Chronic Gastritis classically present with?

A

Vit B12 deficiency

142
Q

What is the role of Enterokinase?

A
  • Present in the brush border in the duodenal lumen -> converts trypsinogen to trypsin
  • > (converts all the other zymogens to their active form -> preventing autodigestion of the pancreas)
143
Q

What enzyme is responsible for stimulating release of zymogens (inc. trypsinogen) from the Pancreas?

A

CCK

144
Q

What 4 things are part of the diagnostic criteria for PBC?

A
  • Auto-mitochondrial antibodies (AMA)
  • Cholestatic picture of raised ALP
  • Raised ESR
  • Liver biopsy
145
Q

Which ion is dipeptide and tripeptide absorption dependant on?

A

H+ ion (using Pep-T1!!)

146
Q

A 72-year-old man has presented to A&E with quite intense, continuous abdominal pain and bleeding from the rectum. He says he has not opened his bowels in 6 days and when you see him on the ward, he says he has been vomiting today and his abdomen is distended. Given this information what is the most likely diagnosis?

a) Small bowel obstruction
b) Gastritis
c) Adhesions
d) Large bowel obstruction
e) Crohn’s disease

A

b) large bowel obstruction
* A bowel obstruction refers to when the passage of food, fluids and gas, through the intestines becomes blocked. Small bowel obstruction is more common than large bowel obstruction.

Obstruction results in a build up of gas and faecal matter proximal to the obstruction (before the obstruction). This causes back-pressure, resulting in vomiting and dilatation of the intestines proximal to the obstruction.

Aetiology
Small bowel (SBO) – adhesions and herniae
Large bowel (LBO) – malignancy, diverticular disease, and volvulus

Features of bowel obstruction include abdominal pain, vomiting, abdominal distension, and absolute constipation

But how can we tell the difference between SBO and LBO?

Pain - commonly in SBO, abdominal pain is described as intermittent and colicky but improves with vomiting, while the pain associated with LBO is continuous.

Vomiting - the vomiting in SBO tends to be more frequent, in larger volumes, and bilious, which is in contrast to vomiting during an LBO, which typically presents as intermittent and feculent when present.

Tenderness - tenderness to palpation is present in both conditions, but with SBO, it is more focal, and with LBO, it is more diffuse.

147
Q

A 53 year old obese man presents to his GP with a dull ache in his right upper quadrant. On examination a smooth liver edge is palpable 2cm below the costal margin. Liver function tests are as follows: Bilirubin 19 (normal range 1-20 µmol/l)ALT 76 (normal range 7-55 U/l) ALP 56 (normal range 25-110 U/L). What is the most appropriate next step in investigating and confirming the likely diagnosis?

a) Fibroscan
b) Abdominal ultrasound
c) Abdominal xray
d) MRCP
e) Abdominal CT

A

b) Abdominal ultrasound
* This patient most likely has non-alcoholic steatohepatitis (NASH) (nb. raised ALT). This could be confirmed with a simple abdominal ultrasound scan. Treatment would be lifestyle change.*

148
Q

A 28 year old student presents with episodes of diarrhoea with mucus and weight loss. She usually complains of tiredness and noticed some ulcers in her mouth. Anti-TTG antibodies were confirmed on tests. What signs are also seen by intestinal endoscopy in this condition?

a) Skip lesions
b) Villous atrophy and Crypt hypertrophy
c) Superficial inflammation
d) Transmural inflammation
e) Fistulas

A

b) Villous atrophy and Crypt hypertrophy
* Villous atrophy and Crypt hypertrophy are key signs of Coeliac.

The other signs are associated with IBD*

149
Q

A 40 year old man presents with intense pain that starts in the right upper quadrant and radiates to the shoulder, he also has a yellow tint in his sclera. What is the most likely diagnosis?

a) Kidney stones
b) Acute pancreatitis
c) Alcoholic hepatitis
d) Cholelithiasis
e) Peptic ulcer

A

d) Cholelithiasis

* Cholelithiasis is associated with intense pain that starts in the RUQ and radiates to the shoulder and jaundice*

150
Q

A 55 year old man presents with a six week history of progressive painless jaundice and weight loss. On inspection, he is jaundiced. Abdominal palpation is normal. What is the most likely diagnosis?

a) Crohn’s disease
b) Diverticular disease
c) Duodenal ulcer
d) Pancreatic cancer
e) Renal carcinoma

A

d) Pancreatic cancer

* Progressive painless jaundice and weight loss indicates pancreatic cancer*

151
Q

How does an anal abscess present?

A
  • Pain
  • Bleeding/discharge
  • Fever
  • Associated with IBD
  • More common in males
152
Q

A 25 year old female presents to the general practitioner with a five week history of passing four bloody loose stools per day, tenesmus and waking up during the night to open her bowels. She also reports left lower abdominal pain and fatigue. She has no past medical history of note and is an ex-smoker. Her observations are in normal range and clinical examination reveals abdominal tenderness in the left lower quadrant with no distension or guarding. There is no evidence of peri-anal disease on rectal examination. Serum total immunoglobulin A and immunoglobulin tissue transglutaminase antibody are negative. Faecal calprotectin is 280µg/g (normal range <150µg/g). She is referred urgently to gastroenterology. Which of the following findings on histology would confirm the most likely diagnosis?

a) Villous atrophy and crypt hyperplasia
b) Crypt abscesses
c) Goblet cells Hyperplasia
d) Inflammation from mucosa to serosa
e) Granulomas

A

b) Crypt abscesses
* The patient’s age, symptoms and positive faecal calprotectin result mean inflammatory bowel disease is the most likely diagnosis.

Although it can be difficult to distinguish between ulcerative colitis and Crohn’s disease based on history and examination alone, features in this case which are more commonly associated with ulcerative colitis include bloody diarrhoea, left sided abdominal pain, absence of peri-anal disease and the risk factor that the patient is an ex-smoker.

Histological features in keeping with a diagnosis of ulcerative colitis include inflammatory changes that do not extend beyond the submucosa, goblet cell depletion and crypt abscesses.

Crypt abscesses are collections of neutrophils within an intestinal crypt.*

153
Q

What is first-line for…

a) a mild UC flair?
b) a mild Crohn’s flair?
c) a severe UC flair?
d) a severe Crohn’s flair?

A

a) Mesalazine (5-ASA)
b) Steroids
c) Steroids - IV Hydrocortisone
c) Steroids - IV Hydrocortisone

True-Love Witts criteria for UC flare severity (based on no. of bowel movements/day):
<4 = mild
4-6 = moderate
>6 = severe (-> requires admission for IV hydrocortisone and full supportive care)
>10 = fulimnant

154
Q

What is the treatment of perianal abscess?

A
  • Surgical incision and drainage
  • Seton insertion
  • Abx -> metronidazole, ciprofloxacin
155
Q

A 28 year old student presents with episodes of diarrhoea with mucus and weight loss. She usually complains of tiredness and noticed some ulcers in her mouth. She says that stopping gluten improved her symptoms. What is the skin rash associated with this condition called?

a) Malar flush
b) Acne
c) Dermatitis herpetiformis
d) Pyoderma gangrenosum
e) Erythema nodosum

A

c) Dermatitis herpetiformis
* - Malar flush = Mitral Stenosis
- Dermatiits Herpetiformis = Coeliac Disease
- Pyoderma Gangrenosum = Crohn’s*

156
Q

Which one of the following is TRUE regarding Glycosidic bonds?

a) Cellulose is a polysaccharide that is formed by linkage of alpha-1,4 glycosidic bonds
b) Pancreatic cellulase hydrolyses the beta-1,4 glycosidic bonds in cellulose
c) Salivary and Pancreatic Amylase are used to break down beta-1,4 glycosidic bonds
d) beta-1,4 glycosidic bonds are found in Starch and Glycogen
e) Alpha-1,4 glycosidic bonds are broken down by Salivary and Pancreatic Amylase

A

e) Alpha-1,4 glycosidic bonds are broken down by Salivary and Pancreatic Amylase
* Salivary Amylase and Pancreatic Amylase are enzymes used to break down alpha-1,4 glycosidic bonds that are found in both Starch and Glycogen.

Cellulose is a polysaccharide that is formed by linkage of beta-1,4 glycosidic bonds which can only be broken down by Cellulase that is found in bacteria.

Pancreatic cellulase does not exist, it is found in bacteria, no vertebrate species expresses cellulase*

157
Q

One of the following is NOT a product of bacterial fermentation of undigested carbohydrate in the colon:

a) Vitamin K
b) Short Chain Fatty Acids
c) Nitrogen
d) Hydrogen
e) Vitamin D

A

e) Vitamin D
* Products of bacterial fermentation of undigested carbohydrate in colon:

[1] Short chain fatty acids (energy source in ruminants)

[2] Vitamin K

[3] Gas (flatus) ‒ nitrogen, CO2, hydrogen, methane, hydrogen sulphide*

158
Q

Which of the following is FALSE regarding the innervation of the GI tract?

a) Myenteric plexus is primarily responsible for peristalsis
b) The Myenteric and Meissner’s plexus make up the Enteric Nervous System
c) Myenteric plexus is found between the longitudinal and circular muscles
d) Meissner’s plexus is found in the submucosal layer of the GI tract
e) Sympathetic innervation of the GI tract is by the Meissner’s plexus

A

e) Sympathetic innervation of the GI tract is by the Meissner’s plexus
* Sympathetic innervation of the GI tract is not by the Meissner’s plexus

The Enteric Nervous System (ENS):
The Myenteric (Auerbach's) plexus is responsible for peristalsis and found between the two muscular layers

The Meissner’s (Submucosal) plexus is responsible for secretions and absorption and found in the submucosal layer

The Autonomic Nervous System can act on the ENS:

  • Sympathetic innervation of the GI tract is by the abdominopelvic splanchnic nerves (greater - T5-T9, lesser - T10-11, least - T12)
  • Parasympathetic innervation of the GI tract is by the vagus nerve (CN X) and the pelvic splanchnic nerves (S2-S4)*
159
Q

An 18 year old female presented one month ago with bloody diarrhoea, abdominal pain and feeling generally unwell. Her bloods showed raised inflammatory markers, and a flexible sigmoidoscopy revealed inflammation and ulceration of the rectum and distal colon. What is the most appropriate treatment for the long-term management of her condition?

a) Infliximab
b) Oral steroid therapy
c) Tacrolimus
d) Mesalazine
e) IV Ciclosporin

A

d) Mesalazine
* This patient has presented with symptoms of mild Ulcerative Colitis with a focus primarily in the distal gastrointestinal tract. Treatment as recommended by NICE is with topical rectal aminosalicylates such as Mesalazine

Think about:

  1. What is the condition?
  2. How severe is it?
  3. First line treatment!*
160
Q

What is the definitive treatment of Achalasia?

A

Myotomy!!
-> due to failure of the LOS to relax due to degeneration of the myenteric plexus

(Medical treatments = Botulinum toxin injections (botox)
Medical therapy can be tried (calcium channel blockers / nitrites) in patients who fail botox therapy or are not suitable surgical candidates)

161
Q

Which of the following is FALSE regarding the Intestinal phase of gastric acid secretion?

a) The presence of acid in the duodenum leads to the release of secretin
b) The intestinal phase is always inhibitory to HCl secretion
c) The presence of peptides in the stomach lumen increases gastrin secretion
d) The intestinal phase is always stimulatory to HCl secretion
e) Cholecystokinin is NOT released in response to the presence of acid

A

d) The intestinal phase is always stimulatory to HCl secretion

*The intestinal phase is always INHIBITORY to HCl secretion:
[C] Intestinal Phase
Presence of Acid in duodenum:
Enterogastric (splanchnic) reflex + Secretin release = ↓ Gastrin secretion and its effect on parietal cells

Presence of Fat/CHO in duodenum:
GIP release = ↓ Gastrin secretion + ↓ Parietal HCl secretion

Presence of Fat/Amino Acids in duodenum
CCK release = Pancreatic zymogen secretion, gallbladder contraction, relaxation of sphincter of Oddi, decreased gastric emptying*

162
Q

What nerve palsy causes an inability to abduct the eye?

A

6th nerve palsy

163
Q

What is the most common micro-organism causing Tonsillitis?

A

Strep Pneumoniae

164
Q

What are the clinical features of a button battery in the ear?

A

nb. needs URGENT removal!!

  • > Tissue Necrosis:
  • black discharge
  • facial swelling
  • pain + fever

-> Septal perforation of tympanic membrane (if facing that way)

165
Q

A 41-year-old man attends the ENT clinic complaining of having difficulty hearing. Weber’s test lateralises to the left ear. Rinne’s test is positive for both ears (air conduction is greater than bone conduction). What type of hearing loss would you suspect from these screening test results?

a) Not possible to determine with these results alone
b) Left-sided conductive hearing loss
c) Right-sided sensorineural hearing loss
d) Right-sided conductive hearing loss
e) Left-sided sensorineural hearing loss

A

c) Right-sided sensorineural hearing loss
* Lateralisation in Weber’s test can be the result of either a conductive hearing loss on the ipsilateral side or a sensorineural loss (or mixed hearing loss) on the contralateral side. Rinne’s test can then help to identify the nature and side of the deficit.

Weber’s test (tuning fork on forehead)

In sensorineural hearing loss, the sound will be louder in the normal ear (quieter in the affected ear).

In conductive hearing loss, the sound will be louder in the affected ear.

Rinne’s test (tuning fork on mastoid process then near ear)

An normal result (Rinne’s positive) is when air conduction is better than bone conduction.

An abnormal result (Rinne’s negative) is when bone conduction is better than air conduction*

166
Q

You are assisting the ENT consultant in theatre to finish a total thyroidectomy and whilst you are suturing and closing up the wound, the consultant asks you to undertake the necessary post-operative checks for this type of operation when the patient wakes up from the anaesthetic. What is the consultant referring to?

a) Thyroid function tests
b) Flexible nasolaryngoscopy
c) Microlaryngoscopy
d) Neck ultrasound

A

b) Flexible nasolaryngoscopy
* TFTs - no point in checking TFTs because you just removed the thyroid, you know it’s not gonna be functional.

Flexible nasolaryngoscopy - Look at their vocal cords due to post-op risk of damage to recurrent laryngeal nerve!

Microlaryngoscopy is not useful because its done when the patient would be asleep

Neck ultrasound wouldn’t have any usefulness*

167
Q

What is the first line investigation for progressive worsening of hoarse voice?

A

Flexible laryngoscopy

to visualise vocal cords and for biopsy!

168
Q

What is the stepwise management for acute epistaxis?

A

1 - Holding the nose and tilting head anteriorly

2 - Insertion of cotton wool soaked in adrenaline solution

3 - Cautery

169
Q

What are the characteristic signs of vestibular neuronitis?

A
  • Vertigo triggered by a recent viral illness

*NOT triggered by head movements -> BPV!

170
Q

You are seeing a 53 year old female with a 2-day history of acute right sided hearing loss. Rinne’s is negative on the left and positive on the right side. Weber’s lateralises towards the left ear. What type of hearing loss does this patient most likely have?

a) Right conductive hearing loss
b) Right sensorineural hearing loss
c) Left conductive hearing loss
d) Left sensorineural hearing loss

A

c) Left conductive hearing loss

171
Q

60 year old lady non-smoker presents to Head & Neck Clinic with 3 month history of hoarse voice. She was unwell earlier in the year with septic shock and multi-organ failure and required intubation in Intensive Care for 6 weeks. Flexible nasal endoscopy reveals a smooth, pink nodule on the posterior aspect of the right vocal cord. What is your diagnosis?

a) Squamous cell carcinoma
b) Chronic laryngitis
c) Vocal cord nodule
d) Vocal cord granuloma

A

d) Vocal cord granuloma
* SCC - unlikely because she is not a smoker

CL - patient not in pain so exclude it

VCN - nodule associated with people who use their voice excessively e.g. teachers, singers, construction workers

VCG - traumatised vocal chord due to intubation up to 6 weeks puts her at risk of granuloma formation*

172
Q

You are seeing Marina, a 19 year old female in the GP surgery who has previously presented with painful lymphadenopathy following an upper respiratory tract infection and was reassured and discharged with a review in two weeks. She noticed that she has lost some weight but she says its because of her appetite. During her review, you palpate 2 x 1cm, round, firm, fixed, non-tender lymph node in the right anterior cervical chain. How will you manage this patient?

a) Urgent 2 week head & neck cancer referral
b) 7 days oral co-amoxiclav
c) CT scan
d) Ultrasound of her neck

A

a) Urgent 2 week head & neck cancer referral
* Stepwise approach:
1. If there is normal lymphadenopathy (oval, smooth, regular, mobile, tender) with no red flags then reassure and review in two weeks
2. If persistent after review then refer to ENT cancer and they would do cytology and biopsy.

  1. If there is an abnormal lymphadenopathy (round, firm, irregular, fixed, non-tender) with red flags (weight loss, bleeding) then urgently refer to ENT cancer and they would do cytology and biopsy

If malignancy confirmed then:

  1. CT scan*
173
Q

What is the initial step in treatment of glue ear?

What should you offer if it persists?

A
  • Observe for 3 months + review
  • > (glue ear is usually self-limiting)
  • If persists: then grommets can be indicated
174
Q

You see a 32 year old female in the GP surgery who presents with an lateral neck lump swelling which has been present for the past 1 week. The neck lump is transilluminated with light and does not move when the tongue is protruded. What is your diagnosis?

a) Branchial cyst
b) Dermoid cyst
c) Thyroglossal cyst
d) Sebaceous cyst

A

a) Branchial cyst
* Branchial cysts present as a round, soft, cystic swelling between the angle of the jaw and the sternocleidomastoid muscle in the anterior triangle of the neck.

This swelling will transilluminate with light, as the fluid in the cyst transmits the light throughout the cyst.

Management:

  • Conservative, without any active intervention, where it is not causing problems
  • Surgical excision where recurrent infections are occurring, there is diagnostic doubt, or it is causing other problems*
175
Q

A 22-year-old female attends A&E with a nosebleed. She is leaning forward and pinching her nose. The nosebleed started 3 minutes ago, and she has been pinching her nose since. Observations are stable. On examination, she is breathing from her mouth, no coughing or choking. There is blood dripping from the right nostril when she takes her hand away. On closer examination, you can see a bleeding site on the right septum. What is the most appropriate next step in management?

a) Direct compression for 10-15 mins
b) Nasal packing
c) Nasal cautery
d) Nasal balloon catheter
e) Transnasal endoscopy with direct cautery

A

c) Nasal cautery
* Start with the least invasive measures first

Stepwise process:

1 - Antiseptic cream (if there is recurrent bleeding but without bleeding points)

2 - Chemical cautery with silver nitrate (if there are bleeding points)

3 - Examination under anaesthesia and electrical cautery (if the previous doesn’t work)*

176
Q

What is the first line investigation for conductive hearing loss?

A

Otoscopy

-> to look for any obstruction

177
Q

What are the cardinal symptoms of Meniere’s disease?

A

1 - Vertigo

2 - Tinnitus

3 - Hearing loss

also: episodic, experience of aural fullness

178
Q

What are the cardinal features of Vestibular Schwannoma?

A
  • Unilateral sensorineural hearing loss
  • Tinnitus
  • Headache in the occipital region
    (-> as tumour is at the cerebellopontine angle -> mass effect)
179
Q

What are the key features of Conjunctivitis?

A
  • Redness
  • Crusty “gritty” eyes
  • Itchiness
  • NO PAIN
180
Q

What would the following symptoms indicate? vertical diplopia, exacerbated when looking downwards and inwards (such as when reading or walking down the stairs) with a head tilt.

A
  • CN 4 palsy
  • Trochlear nerve palsy commonly presents with vertical diplopia, exacerbated when looking downwards and inwards (such as when reading or walking down the stairs). Patients can also develop a head tilt away from the affected side*
181
Q

A 67-year-old man presents with a sensation of the room spinning around when he lies down, especially when reclining on his left-hand side. This has been occurring for around a month and is getting worse. There is no change in his hearing and no tinnitus. Examination of the ears is normal. He has nystagmus to the left when the Hallpike manoeuvre is performed. He has reduced lateral flexion of the neck. Which of the following is the most appropriate management plan?

a) Arrange urgent MRI of cerebellopontine angle
b) Perform Epley’s manoeuvre
c) Prescribe a course of vestibular sedatives
d) Refer to physiotherapy
e) Refer to vestibular rehabilitation

A

b) Perform Epley’s manoeuvre
* in BPPV, vertigo can be seen without association with hearing loss or tinnitus. In addition to that, during the Dix-Hallpike manoeuvre there was nystagmus seen.

These two signs point to the diagnosis of BPPV which is managed by performing Epley’s manoeuvre*

182
Q

What CN palsy is indicated by this: “Right eye turns downward and outwards. Unable to adduct right eye. Also Ptosis and dilatation of right eye.”

A

Right CN III palsy

183
Q

A 5-year-old boy with recurrent attacks of otitis media is suspected of developing otitis media with effusion (glue ear). Which of the following findings is most consistent with unilateral middle ear effusion?

a) No lateralization with Weber’s test
b) Negative Rinne’s test on the ipsilateral side
c) Negative Weber’s test only on the contralateral side
d) Lateralisation with Weber’s test to the unaffected ear
e) Positive Rinne’s test on the ipsilateral side

A

b) Negative Rinne’s test on the ipsilateral side
* Middle ear effusion is a cause of conductive hearing loss. A negative Rinne’s test would indicate that bone conduction > air conduction and therefore likely conductive hearing loss on the affected side.*

184
Q

What is the clinical presentation of a Cholesteatoma?

What is the treatment of Cholesteatoma?

A
  • Persistent, intermittent, offensive, foul smelling discharge (long history), headache and otalgia
  • Otoscopy: area of white in the attic behind the tympanic membrane
  • nb. it is not related to cholesterol or tumours!*
  • Treatment: Mastoidectomy!!
185
Q

A 55-year-old man presents to his General Practitioner (GP) with significant hearing loss in his right ear. He reports that it has been present since the morning and that it was preceded by a ‘popping’ sensation in the same ear. On questioning, he notes that he had mild flu-like symptoms a week ago; however, they soon settled and he has not previously had in problems with his hearing. On examination, gross hearing is reduced in the right ear. Rinne’s test is positive bilaterally and Weber’s test lateralises to the left. Which of the following is the next best step in management?

a) Routine referral to audiometry
b) Provide safety-net advice and review in one week
c) Prescribe acyclovir
d) Urgent same-day referral to ENT
e) Prescribe gentamicin and hydrocortisone ear drops

A

d) Urgent same-day referral to ENT

* sudden loss of hearing = red flag!*

186
Q

What is the investigation and treatment of BPPV?

A
  • Investigation: Dix-Hallpike maneouvre

- Treatment: Epley’s maneouvre

187
Q

What is the pathophysiology of Atopic Dermatitis?

A

Type I HS reaction!!

IgE mediated

188
Q

in which sites does atopic dermatitis occur

A

Flexural sites:

-> ie. inside elbows, behind the knees, in front of neck

189
Q

What is first line treatment of Impetigo?

A

Antiseptic (hydrogen peroxide) cream

190
Q

What is the causative organism of Impetigo?

A

One or both of:

  • Group A Strep (Strep Pyogenes)
  • Staph Aureus
  • Both are gram-positive cocci*
191
Q

How can you differentiate between DRESS and SJS/TEN reaction?

A
  • both severe drug reactions characterised by an extensive skin rash*
  • DRESS: without erosion and blistering of the skin
  • SJS/TEN: with erosion and blistering of the skin
192
Q

What is first line treatment for Seborrhoeic dermatitis?

A
  • Ketoconazole shampoo
193
Q

Which areas does Seborrhoeic dermatitis affect?

A
  • Scalp
  • Nasolabial folds
  • Eyebrows
194
Q

What is the causative organism of Seborrhoeic dermatitis?

A

Malassezia yeast

195
Q

What are the clinical features of Seborrhoeic dermatitis?

A
  • Erythema
  • Dermatitis
  • Crusted dry skin on the scalp
196
Q

All the following can be used in the treatment of atopic eczema EXCEPT:

a) Topical steroids
b) Antihistamines
c) Emollients
d) Antifungal cream

A

d) Antifungal cream

197
Q

A 50 year old male, previous smoker, has recently undergone renal transplant. He presents with an aggressive-looking lesion on his right cheek. What is the most likely diagnosis?

a) Squamous cell carcinoma
b) Basal cell carcinoma
c) Malignant melanoma
d) Cutaneous lymphoma

A

a) Squamous cell carcinoma
* in immunocompromised renal transplant recipients (RTRs), the incidence and aggressiveness of skin cancers, particularly squamous cell carcinome, increase by up to 100-fold!*

198
Q

Actinic keratosis cannot be treated by:

a) Curettage
b) Topical steroid
c) Cryotherapy
d) Topical imiquimod

A

b) Topical steroid

* Topical steroid does not work on Actinic Keratosis -> but can use topical Fluorouracil!*

199
Q

What is the key difference between Acne and Rosacea?

A

There are no comedones in Rosacea!

200
Q

What skin condition is: “golden crusty skin”?

A

Impetigo!!

201
Q

What is the treatment of Impetigo?

A
  • Localised: topical abx treatment

- Widespread, severe: oral abx

202
Q

What type of testing is used for:

a) Type 1 HS reaction in the skin
b) Type 4 HS reaction in the skin

A

a) Skin prick testing or Specific IgE levels

b) Skin patch testing

203
Q

What is SCORTEN?

A

Severity of illness score used to predict in-hospital mortality for for SJS/TEN

204
Q

A 56 year old man presented with multiple, well-defined, warty and greasy papules with a “stuck on” appearance on his back. What is the most likely diagnosis?

a) Malignant melanoma
b) Seborrhoeic keratosis
c) Melanocytic nevi
d) Cherry angioma

A

b) Seborrhoeic keratosis
* Well-defined, warty and greasy papules with a “stuck on” appearance = Seborrhoeic keratosis

Acute onset of lots of these bad boys is an indication of malignancy! That’s called Leser-Trelat sign.

NB. multiple, well-defined warty papules on the palms = viral warts (contagious!)*

205
Q

A 35 year old female presents with multiple, well-defined and warty papules on his palms. What is the causative organism?

A

HPV!!

Viral warts = contagious!!

206
Q

A 22 year old man presented with severe erythema, redness, and crusted dry skin on his scalp, nasolabial folds and eyebrows. Antifungal creams and Ketoconazole shampoo were commenced but they were unresponsive. These signs of severeness and unresponsiveness are commonly an indication for:

a) HIV testing
b) Skin prick testing
c) IV antibiotics
d) Skin patch testing

A

a) HIV testing

* unresponsive, severe Seborrhoeic Dermatitis is an indication for HIV testing!!*

207
Q

A 14 year old girl presents with small itchy lumps on her forearm and chest after hugging a tree. She says that the rash is spreading and she feels swelling underneath her skin. What treatment is most appropriate?

a) Topical antibiotics
b) Antivirals
c) Topical steroids
d) Antihistamines
e) Topical retinol

A

d) Antihistamines

208
Q

A 32 year old patient presents with a history of generalised patches of erythema with thin and wrinkled skin that changed into fluid filled blisters. He is unwell, feverish, and appears to be dehydrated. What is the best appropriate approach to this patient?

a) Primary care management
b) Admission and IV resuscitation
c) Give topical steroids
d) Give topical antibiotics
e) Reassure and review in a week

A

b) Admission and IV resuscitation
* Patient presenting with symptoms of Staphylococcal Scalded Skin Syndrome

Most patients will require admission and treatment with IV antibiotics.

Fluid and electrolyte balance is key to management as patients are prone to dehydration.*

209
Q

What are the key clinical features of normal pressure hydrocephalus?

A

Triad of:

  • Gait disturbance
  • Urinary incontinence
  • Cognitive decline
210
Q

A 20 year old man is assaulted while watching a football match. On arrival at the local Emergency department he opens his eyes to pain, says inappropriate words and localises to pain. What is his Glasgow coma score?

a) 6
b) 7
c) 8
d) 9
e) 10

A

e) 10

E2/4 - open to pain

V3/5 - inappropriate words

M5/6 - localised to pain

GCS 10/15

211
Q

What is the GCS scoring system?

A

E (Eye-opening response):
• Spontaneous - open with blinking at baseline = 4
• To verbal stimuli, command, speech = 3
• To pain only (not applied to face) = 2
• No response = 1

V (Verbal response):
• Oriented = 5 
• Confused conversation, but able to answer questions = 4 
• Inappropriate words = 3 
• Incomprehensible speech = 2 
• No response = 1 

M (Motor Response):
• Obeys commands for movement = 6
• Purposeful movement to painful stimulus = 5
• Withdraws in response to pain = 4
• Flexion in response to pain (decorticate posturing) = 3
• Extension response in response to pain (decerebrate posturing) = 2
• No response = 1

GCS = out of 15

212
Q

What are the key features of trigeminal neuralgia?

A
  • Sudden onset of severe facial pains (“like an electrical shock”) across the jaw -> usually lasts for 5-10s
  • Triggered by cold winds, shaving (touch), chewing
213
Q

What is the treatment of trigeminal neuralgia?

A
  • 1st line: Carbamezapine

- Surgical: Microvascular decompression

214
Q

What is the treatment of Cluster headaches?

A

Triptans (ie. sumatriptan) + High-flow oxygen

215
Q

What is the treatment of Acute migraine attacks?

A

Triptans (ie. sumatriptan)

216
Q

What is the prophylactic treatment of migraines?

A

Propranolol and Topiramate

217
Q

What CN does examination of “shutting the eyes” test?

A

CN VII

218
Q

What CN does examination of the “corneal reflex” test?

A
  • afferent = CN V1

- efferent = CN VII

219
Q

What CN does examination of the “vestibulo-occular reflex” test?

A
  • afferent = CN VIII

- efferent = CN III

220
Q

What CN does examination of the “pupillary response” test?

A
  • afferent = CN II

- efferent = CN III

221
Q

Disorders of which part of the brain cause Nystagmus?

A

Cerebellum

222
Q

What is the treatment of cluster headaches?

A

First line:

  • Triptans - Sumatriptan injections
  • High-flow (100%) O2
  • Transcutaneous Vagus Nerve Stimulation (TVNS)

Surgical:
- Stimulation device implant

223
Q

What is the prophylaxis treatment for cluster headaches?

A
  • Verapamil

If that doesn’t work:

  • Lithium
  • Prednisolone
  • Occipital Nerve block
224
Q

What are the typical clinical features of cluster headaches?

A
  • Excruciating attacks of pain in one side of the head, often felt around the eye (ie. red, watering eye)
  • Often v quick onset without warning: severe, burning or piercing -> tends to affect the same side for each attack
  • The attacks generally last between 15 minutes and 3 hours, and typically occur between 1 and 8 times a day
  • Occurs in bouts of days-weeks, before a period of remission (months or years), before starting back up again
225
Q

What are the characteristic clinical features of Cauda Equina Syndrome?

A
  • commonly affected nerve = S2-S4*
  • Severe back pain
  • Urinary retention
  • Numbness around the perineum
  • Absent ankle jerks
226
Q

What are the characteristic clinical features of Guillain-Barre Syndrome?

A
  • Acute, symmetrical, ascending weakness
  • > (muscle weakness starts in the legs and spreads to the arms)
  • Typically occurs after a vial infection
227
Q

What are the characteristic clinical features of Gerstmann’s Syndrome?

A

affects the Parietal Lobe (specifically: left parietal lobe at the level of the angular gyrus)

  • Agraphia
    (can’t write)
  • Acalculia
    (can’t do simple maths)
  • Left-right confusion
    (can’t tell left from right)
  • Finger Agnosia
    (can’t identify fingers)
228
Q

What are the differing clinical features of L3 and S1 radiculopathy?

A
  • Unilateral leg pain and numbness (like sciatica) - affects certain reflexes*
  • L3 = absent patellar reflex
  • S1 = absent ankle jerks
229
Q

A 65 year old man presented to his general practitioner (GP) with difficulty initiating walking. His GP suspects Parkinson’s disease. Which of the following is NOT an essential treatment of Parkinson’s disease?

a) Levodopa
b) Amitriptyline
c) COMT inhibitors
d) Dopamine agonists
e) MAO-B inhibitors

A

b) Amitriptyline
* Amitriptyline is not essential to treat the symptoms caused by Parkinsonism (Rigidity, Resting Tremor, Bradykinesia), but it can be used to treat depression which is commonly diagnosed in Parkinsonism.*

230
Q

What clinical features are associated with GCA?

A
  • Tenderness around the temporal artery
  • (tenderness when brushing hair)
  • Associated with vision changes
231
Q

What clinical features are associated with Tension headache?

A
  • Ache around the temples and forehead

- Not severe, but uncomfortable

232
Q

What clinical features are associated with Migraine?

A
  • Photophobia
  • Phonophobia
  • ?possible Aura
  • Usually unilateral
  • Relieved by sitting in a dark room for 20-30mins
233
Q

What clinical features are associated with Sinusitis?

What is it’s treatment?

A
  • Headache over the upper portion of the face, cheekbones, eye and forehead
  • Usually after a viral infection

(self-limiting -> ie. no treatment)

234
Q

A 50 year old right handed barman presents with cognitive language dysfunction. He has difficulty with language comprehension and difficulty reading e-mails. For the last six weeks he has been taking dihydrocodeine (oral opiates) for rib pain. Which of the following is the most likely explanation for these findings?

a) Alcohol intoxication
b) Lesion in left temporo-parietal area
c) Opiate dependence
d) Right chronic subdural haematoma
e) Tumour in right inferior frontal lobe

A

b) Lesion in left temporo-parietal area
* Language comprehension and difficulty reading emails

The vast majority of people (>90%) have language in their left hemisphere and they are right-handed

If you are left-handed you still have a high chance (60%) of having language in your left hemisphere.

20% of left handed people will have bilateral language hemispheres and the last 20% will have right hemisphere dominant language.

Expression of language is in Broca’s area located in areas 44 and 45

Comprehension of language is in Wernicke’s area around the superior temporal gyrus*

235
Q

What are the clinical features of SAH?

A
  • Sudden-onset severe (“thunderclap”) headache
  • > (unlike anything they’ve experienced before)
  • > usually occipital in origin
  • Neck stiffness
  • Feeling and being sick
  • Photophobia
  • Blurred or double vision
236
Q

What is the first line investigation for SAH?

What should you do next?

Then what should you do if diagnosis of SAH is confirmed?

A
  • CT head
  • Lumbar puncture -> to look for xanthochromia and red cell count
  • Angiography -> to locate the cause
237
Q

What are the clinical features of an ACA ischaemic stroke?

A
  • Contralateral Lower Limb weakness
238
Q

What are the clinical features of an MCA ischaemic stroke?

A
  • Hemiparesis of lower contralateral face
  • Speech impairment
  • Contralateral weakness
239
Q

What are the clinical features of a PCA ischaemic stroke?

A
  • Acute vision loss

- Memory loss

240
Q

A 16 year old boy presented with a spinal cord injury with absent motor response below C7. Which of the following is an example of a descending spinal cord tract?

a) Anterior spinocerebellar tract
b) Dorsal column pathway
c) Lateral corticospinal tract
d) Lateral spinothalamic tract
e) Posterior spinocerebellar tract

A

c) Lateral corticospinal tract

241
Q

A 60-year-old man comes in with dysarthria, dysphagia and regurgitation of fluids which can sometimes result in choking. When he talks to you, his speech is quite nasal and hoarse. On examination, his tongue is flaccid, and jaw jerk is absent. His sensation is intact and normal. What is the most likely diagnosis?

a) Cervical spine lesion
b) Motor neurone disease
c) Multiple sclerosis
d) Progressive bulbar palsy
e) Bell’s Palsy

A

d) Progressive bulbar palsy
* Dysarthria, dysphagia, regurgitation, flaccid tongue, absent reflexes = all signs of a bulbar palsy presenting with LMN features :)

The patient presents with the signs of a bulbar palsy. This is caused by dysfunction in cranial nerves 9-12 from their origin in the medullary nuclei to their termination in the muscles supplying the tongue and the muscles coordinating speech and swallow. It is a ‘lower motor neurone’ lesion of the bulbar cranial nerves.

Pseudobulbar palsy, an UMN lesion affecting the corticobulbar tracts, would present with the other symptoms: emotional lability (uncontrollable laughing or crying), ‘hot potato’ speech (slow, thick speech), brisk reflexes.*

242
Q

What is the key diagnostic finding to confirm a diagnosis of GCA?

A
  • Raised ESR
243
Q

Multiple sclerosis is an autoimmune demyelinating condition affecting the CNS. What cells are targeted?

A
  • OLIGODENDROCYTES!!!

NOT Schwann cells!!!!

244
Q

Which NT is affected in Parkinson’s disease?

A

Dopamine

245
Q

A 75 year old woman is brought into the emergency department following a fall down 2 flights of stairs. She is opening her eyes to voice. She fails to answer your questions but is calling out random words and is unable to form a sentence. She moans and withdraws from painful stimuli. What is her GCS score?

a) 6
b) 7
c) 8
d) 9
e) 10

A
  • 10
  • The patient’s GCS is 10: E3 V3 M4. She is opening her eyes to voice only therefore score 3/4. She is speaking, but her words are inappropriate as she is unable to form sentences, therefore scores 3/5. She is only able to withdraw from pain and therefore scores 4/6.*
246
Q

A 29-year-old woman presents to her General Practitioner with a four-month history of headaches. She explains that they are preceded by paraesthesiae spreading up one arm over half an hour, with the headache consisting of a left-sided throbbing associated with nausea. She has no past medical history. Her doctor suspects migraines. Which medication is first-line prophylaxis for migraines?

a) Gabapentin
b) Topiramate
c) Propranolol
d) Sumatriptan
e) Paracetamol

A

c) Propranolol

* Topiramate is used if Propranolol is contra-indicated!!*

247
Q

A middle aged man presents with a history suspicious of subarachnoid haemorrhage. You assist your registrar in performing a lumbar puncture. You are asked to label the bottles in the order that they are collected and to send them off. Other than normal cell microscopy, what else would it be important for you to send the samples for?

a) Erythrocyte sedimentation rate (ESR)
b) Culture and sensitivity
c) Xanthochromia
d) Oligoclonal bands
e) India ink staining

A

c) Xanthochromia

    • red cell count + xanthochromia suggest SAH!!
  • culture and sensitivity is only used if meningitis (headache, photophobia, neck stiffness) is the main differential (but the stem said that SAH is the main differential)*
248
Q

What is first line treatment of GCA?

A

Prednisolone

249
Q

A 69 year old man presented with left sided neglect. Which of the following structures is most likely to be affected?

a) Left frontal lobe
b) Left parietal lobe
c) Right frontal lobe
d) Right parietal lobe
e) Right temporal lobe

A

d) Right parietal lobe
* Left sided neglect indicates a right hemisphere lesion, right parietal lobe.

Others symptoms might include constructional apraxia (inability to draw a 5-pointed star), loss in familiar environments, and dressing apraxia (inattention to the left side when dressing)*

250
Q

A 70-year-old man comes into your clinic with history of seizures. His wife attends with him and describes he lost awareness and was confused when he came round. During the period of lost awareness, she describes he was fiddling with the buttons on his jacket and was smacking his lip. What is the most likely diagnosis?

a) Frontal lobe simple partial seizure
b) Frontal lobe complex partial seizure
c) Temporal lobe complex partial seizure
d) Temporal lobe simple partial seizure
e) Occipital lobe partial seizure

A

c) Temporal lobe complex partial seizure
* Impairment in consciousness = complex, presence of clinical features such as fiddling, lip smacking, these are known as automatisms, which are commonly features of temporal lobe seizures

Clinical features of Temporal lobe epilepsy (TLE)

Seizure auras occur in many TLE patients. Auras are in fact simple partial seizures and can occur in isolation but occur in the majority of patients at the onset of a complex partial seizure. They can last from seconds to a long as 1-2 minutes before consciousness is lost.

Auras observed in patients with TLE may manifest with:

  • Viscerosensory symptoms such as a rising epigastric sensation and experiential phenomena such as fear, déjà and jamais vu, visceral and auditory illusions
  • Complex auditory or visual hallucinations
  • Gustatory and olfactory hallucinations

Automatisms are common clinical features of temporal lobe seizures representing involuntary motor activity e.g. lip smacking, fidgeting, fiddling, chewing, swallowing, fumbling.. etc.*

251
Q

A 65 year old man presented to his general practitioner (GP) with difficulty initiating walking. His GP suspects Parkinson’s disease. What issues need to be considered when using Levodopa?

a) Dyskinesias
b) Postural hypotension
c) Atrial fibrillation
d) Nausea
e) Hallucinations

A

a) Dyskinesias

252
Q

A 65 year old man presented to his general practitioner (GP) with difficulty initiating walking. His GP suspects Parkinson’s disease. Which of the following symptoms are also associated with Parkinson’s disease?

a) Rapid progression
b) Symmetrical rest tremor
c) Lack of rest tremor
d) Rigidity
e) Poor response to treatment

A

d) Rigidity
* Likely to be Parkinsonism if:

— Bradykinesia +≥ 1 tremor, rigidity, postural instability

— No other cause / atypical features

— Slowly progressive (> 5-10 yrs)

— Supported by asymmetric rest tremor, good response to dopamine replacement treatment

Less likely to be Parkinsonism if:

  • rapid progression
  • symmetrical
  • lack of rest tremor
  • poor response to treatment
  • early falls
  • early dementia (dementia w lewy bodies*
  • other abnormal neurological signs*
253
Q

A 28-year-old man is brought into A&E with a reduced level of consciousness. His friends believe he has ingested large quantities of an unknown substance. He opens his eyes briefly on pressure on the fingertips, but not in response to voice. He is making moaning sounds but no words or sentences. On assessment of his motor response, there is abnormal flexion in response to pain, with no localisation. What does this patient score on the Glasgow Coma Scale (GCS)?

a) 8
b) 6
c) 9
d) 5
e) 7

A

e) 7
* Opens eyes briefly on pressure but not to voice = E2/4

Moaning sounds but no words or sentences = V2/5

Abnormal flexion in response to pain = M3/6

GCS = 7/15*

254
Q

What is the differential between peroneal nerve palsy and L5 radiculopathy?

A
  • both = foot drop*
  • Peroneal nerve palsy: medial leg and foot loss of sensation, inversion is unaffected, but ankle eversion is weak
  • L5 radiculopathy: lateral leg and foot loss of sensation, ankle eversion is unaffected, but inversion is weak
255
Q

Which key clinical feature would suggest Vascular Dementia?

A
  • Stepwise progression of dementia
256
Q

Which key clinical features would suggest Lewy body Dementia?

A
  • Parkinsonism

- Visual hallucinations

257
Q

Which key clinical feature would suggest CJD Dementia?

A
  • Deteriorating rapid progression of dementia
258
Q

A 21 year old patient presents to General Practice after having a seizure. Whilst undergoing an initial assessment, he has another seizure that lasts more than five minutes. A nurse has attempted to obtain intravenous access but has had a number of unsuccessful attempts. What is the most appropriate initial treatment?

a) Urgent CT head scan
b) Urgent intubation and ventilation
c) Intravenous lorazepam once IV access is obtained
d) Buccal midazolam
e) Rectal temazepam

A

d) Buccal midazolam
* Status epilepticus is a medical emergency, the first line treatment for seizures and status epilepticus is benzodiazepines. Since intravenous access is yet to be obtained, the most appropriate initial treatment in this patient is buccal midazolam

All other options are initially inappropriate because this is an emergency, attempts for IV access would delay treatment.

Temazepam is not used in treatment of seizures*

259
Q

What is the definition of Status Epilepticus?

A
  • A seizure that lasts longer than 5 minutes, or having more than 1 seizure within a 5 minutes period, without returning to a normal level of consciousness between episodes
  • Medical Emergency!!
260
Q

What is the first-line treatment of Status Epilepticus?

A
  • Medical Emergency!!

- IV Lorazepam

261
Q

Concerning the motor homunculus. Which of the following structures is represented the most laterally?

a) Eye
b) Face
c) Tongue
d) Knee
e) Wrist

A

c) Tongue

* Knee is most medial*

262
Q

What is located most medially and most laterally in the sensory homunculus?

A
  • Most medially: genitals and lower extremities

- Most laterally: intra-abdominal

263
Q

Which body part has the largest representation in the sensory homunculus?

A

Lips

264
Q

Which body part has the largest representation in the motor homunculus?

A

Hands

265
Q

A 28-year-old-man presents to the Emergency Department complaining of a sudden-onset occipital headache associated with vomiting. His symptoms started 2 hours ago and are continuing. He has a previous history of infrequent migraine without aura, which also causes nausea but not vomiting. His current headache is much more severe than his usual migraine, which has not previously started suddenly. Examination is unremarkable. In particular, there is no neck stiffness or photophobia. Which one of the following management options would be the most appropriate?

a) CT brain scan, followed by lumbar puncture 12 hours post headache if CT normal
b) A diagnosis of migraine and referral to a neurologist
c) CT brain scan followed by angiography
d) Immediate lumbar puncture to exclude meningitis and subarachnoid haemorrhage
e) CT brain scan, followed by discharge if normal

A

a) CT brain scan, followed by lumbar puncture 12 hours post headache if CT normal

266
Q

What are the key clinical features that Frontotemporal Dementia associated with?

A
  • Early changes in personality and behaviour
  • > increasingly inappropriate social behaviour, loss of empathy and other interpersonal skills (ie. having sensitivity to another’s feelings)
  • Early dysphasia
  • Preservation of memory
267
Q

What are the key clinical features that Alzheimer’s is associated with?

A
  • Hippocampus*
  • Early memory disturbance
  • Language
  • Visuospatial problems
268
Q

A 51 year old man is noted to have a left trigeminal nerve palsy. Which of the following is the most likely examination finding?

a) He has absent taste sensation to the anterior two thirds of the left of the tongue
b) He has numbness over the left mandible
c) He has numbness over the right maxilla
d) He has numbness over the right posterior aspect of the scalp
e) He is unable to smile with the left side of his mouth

A

b) He has numbness over the left mandible

269
Q

A 30-year-old woman is referred to the Neurology clinic and has been diagnosed with Epilepsy after clinical observation and Electroencephalography. Her seizures occur for around one minute with loss of consciousness and her husband describes her body as stiffening up, followed by rhythmic limb jerking. After the episodes she usually experiences a post-ictal phase. She has no other medical history and is currently 2-months pregnant. Which of the following is the most appropriate pharmacotherapy?

a) Natalizumab
b) Sodium Valproate
c) Lamotrigine
d) Lorazepam
e) Carbamazepine

A

c) Lamotrigine
* Woman has tonic-clonic seizures: Sodium Valproate would usually be first-line BUT this woman is pregnant, so safest option would be Lamotrigine! (bc S.V is a teratogen!)*

270
Q

What is the first-line treatment for tonic-clonic, myoclonic and absence seizures?

What about status epilepticus?

A
  • Sodium Valproate
    (nb. Lamotrigine would be used if pt. was preggo!)
  • IV Lorazepam
271
Q

A 30 year old man presents to his GP as he is “unable to cope with one more of these headaches.” He says he has had ten in the last week. They last around 30 minutes each, and during these headaches, which focus around the right eye, he says he experiences ipsilateral lacrimation. Neurological examination shows no physical abnormalities. What is the most likely diagnosis?

a) Meningitis
b) Migraine
c) Cluster headaches
d) Tension headache
e) Raised intracranial pressure

A

c) Cluster headaches
* classic presentation of cluster headaches, with frequent, short-lasting episodes of periorbital pain with additional clinical features, in this case, lacrimation.*

272
Q

What investigation would you do to reach a definitive diagnosis of GCA?

A

Temporal Artery biopsy

273
Q

A 32 year old women presents to her GP with headaches. She is a secretary and the headaches frequently come on while at work, and can be up to 6/10 in severity. They are usually focused across he forehead and around the sides of her head in a symmetrical fashion. They usually resolve over 30 minutes or so. She has no visual changes and rarely gets the headaches when not at work. She is concerned about a brain tumour. What is the most appropriate management?

a) Oxygen
b) Codeine
c) Reassurance
d) Prednisolone
e) Sumatriptan

A

c) Reassurance
* classic tension headache -> can suggest simple analgesics ie. paracetemol, ibruprofen, but it resolves <30 mins, so debatable*

274
Q

A patient is brought into the emergency department following a road traffic accident. He was the front seat passenger of a car in a head on collision traveling at 60mph. He is opening his eyes to speech, complaining loudly of pain in his right lower limb (which has sustained an open tibial fracture) and cannot concentrate on questions being asked. He is asking for his friend and cannot identify where he is. He intermittently follows your command to squeeze your fingers with his left hand but cannot do the same with his right. What is his GCS score?

a) 13
b) 15
c) 14
d) 11
e) 12

A

a) 13
* In the above scenario, the patient is not opening eyes spontaneously, hence scores E3/4. He is confused, hence scores V4/5. He is able to follow commands, even if intermittently and unilaterally, therefore scores M6/6.*

275
Q

What is first line for cluster headache prevention?

A

Verapamil

276
Q

Which of these medications is not a prophylaxis treatment for migraines?

a) Propranolol
b) Ibuprofen
c) Amitriptyline
d) Topiramate
e) Botulinum toxin type A

A

b) Ibuprofen
* Ibuprofen is not used as a prophylactic measure for migraines.

Prophylaxis with Propranolol (Contraindicated: asthma), Topiramate or Amitriptyline

Acute migraine attack is managed with oral triptan e.g. Sumatriptan

Botulinum toxin type A is a specialist treatment for migraine. To be eligible for this treatment you must have headaches for 15 or more days each month*

277
Q

Which type of amnesia is seen if only the thalamus is damaged and the hippocampus is spared?

A

Retrograde amnesia

Illustrates that memory requires ability to form (hippocampus), store (cerebrum) AND search (thalamus) our memories

278
Q

The medial tracts coming from the brainstem that control posture and locomotion include:

A

Vestibulospinal and tectospinal tract

Ventromedial pathways → under brainstem control

279
Q

What is the function of the semi-circular canals?

A

Detection of rotational acceleration

280
Q

What is tonic labarynthine reflex?

A

A reflex that keeps the axis of the head in a constant relationship with the rest of the body.
→ Uses information from maculae and neck proprioceptors

281
Q

Which one of the following is a feature of A-delta fibres?

a) Largest myelinated
b) 30-70 m/sec velocity
c) Transmission of fast pain and cold
d) 0.5-2 m/sec velocity
e) Transmission of slow pain and warmth

A

c) Transmission of fast pain and cold

  • A-delta fibres:
  • Anatomy: small myelinated
  • Velocity: 5-30 m/sec
  • Function: cold, fast pain, pressure*
282
Q

Which of the following is TRUE about hair cells?

a) The hair cells are embedded in the utricle
b) They directly synapse with the facial nerve
c) The hair cells are embedded in the cupula
d) They detect horizontal acceleration
e) The hair cells are distorted by movement of perilymph

A

c) The hair cells are embedded in the cupula
* Hair cells are embedded within the gelatinous cupula and synapse directly with the sensory neurones of the vestibular nerve*

283
Q

Which of the following is FALSE regarding the autonomic innervation of the male genitalia?

a) Contraction of smooth muscle of urethra causes ejaculation
b) Ejaculation is mediated by a1 receptors
c) Contraction of smooth muscle of urethra causes erection
d) Erection is mediated by b2 receptors
e) Relaxation of smooth muscle of corpus cavernosum causes erection

A

a) Contraction of smooth muscle of urethra causes ejaculation

*Sympathetic effect on the reproductive tract:
Noradrenaline activates a1 receptors on smooth muscle of urethra
→ Contraction of smooth muscle and causes Ejaculation

Parasympathetic effect on the reproductive tract:
Acetylcholine activates muscarinic receptors on smooth muscle of corpus cavernosum
→ Relaxation of smooth muscle & causes Erection*

284
Q

What are the corpus nucleus and putamen collectively known as?

A

Corpus striatum

285
Q

Lateral tracts coming from the motor cortex that control voluntary movements of distal muscles include:

A

Corticospinal and Rubrospinal tract

Lateral tracts coming from the motor cortex control voluntary movements of distal muscles
→ i.e. Corticospinal/Pyramidal tract and the Rubrospinal tract

286
Q

Which of the following is FALSE about the motor loop from cortex - basal ganglia - cortex?

a) The direct pathway of the motor loop acts as a positive feedback loop
b) The indirect pathway of the motor loop acts as an antagonizer to the direct pathway
c) The direct pathway by exciting globus pallidus which causes an excitatory effect on the thalamus
d) The direct pathway of the motor loop functions by selecting specific motor actions
e) The indirect pathway of the motor loop functions by suppressing competing/inappropriate actions

A

c) The direct pathway by exciting globus pallidus which causes an excitatory effect on the thalamus
* The direct pathway of the motor loop acts as a positive feedback loop that initiates the GO signal to the SMA in cortex via inhibiting globus pallidus which causes disinhibitory effect on the thalamus*

287
Q

What is Orexin produced by?

What is it required for?

A
  • Hypothalamus

- Wakefulness

288
Q

What is Short-term memory associated with?

A
  • Reverberating circuits

- When deemed significant this reverberation results in consolidation of the memory into long-term memory storage.

289
Q

What is Intermediate long-term memory associated with?

A

Chemical adaptations at the presynaptic terminal

290
Q

What is Long-term memory associated with?

A

Structural changes in synaptic connections

291
Q

Which type of memories decay the fastest and the slowest?

A
  • Fastest = Visual memories

- Slowest = Auditory memories

292
Q

What are parasympathetic effects on BVs?

A

Usually no effect, except in genitalia and salivary glands

293
Q

Which fibres mediate proprioception and are found in muscle spindles and golgi tendon organs plus the dorsal column pathway?

a) A-alpha fibres and C fibres
b) A-alpha fibres and A-beta fibres
c) A-beta fibres and C fibres
d) A-delta fibres and C fibres
e) A-delta fibres and A-beta fibres

A

b) A-alpha fibres and A-beta fibres

294
Q

Which type of pain is described by the following: ‘‘pain caused by a lesion or disease of the somato-sensory nervous system’’

a) Nociceptive pain
b) Nociplastic pain
c) Functional pain
d) Neuropathic pain
e) Inflammatory pain

A

d) Neuropathic pain

295
Q

Which area of the brain is the Posterior Parietal cortex located in?

What is its function?

A
  • Area 5 and 7
  • A mental image of body in space is generated by somatosensory, proprioceptive and visual inputs to posterior parietal cortex located in areas 5 and 7
296
Q

What structure produces Melatonin?

A

Produced by the Pineal gland under the control of the Suprachiasmatic Nuclei (SCN)

297
Q

What is the function of the Saccule?

A

Detection of vertical acceleration

298
Q

Which of the following is TRUE about the otolith crystals?

a) Backward tilt moves the otolith away from the kinocilium
b) Backward tilt causes hyperpolarisation and decreased discharge
c) Otoliths are affected less by gravity compared to endolymph
d) Forward tilt moves the otolith away from the kinocilium
e) Forward tilt causes depolarisation and increased discharge

A

d) Forward tilt moves the otolith away from the kinocilium
* Otoliths are affected more by gravity compared to endolymph

Backward tilt moves the otolith in the direction of the kinocilium
causing depolarisation and increased discharge of APs.

Forward tilt moves the otolith away from the kinocilium
causing hyperpolarisation and decreased discharge of APs.*

299
Q

Which area of the brain is the supplementary motor area and the premotor area located in?

A

Area 6

300
Q

Which of the following is TRUE about the macula?

a) Vertical forces are detected by the macula in the utricle in the vertical plane
b) Tilt of the head is detected by the macula in the saccule in the horizontal plane
c) Vertical forces are detected by the macula in the utricle in the horizontal plane
d) Tilt of the head is detected by the macula in the utricle in the vertical plane
e) Vertical forces are detected by the macula in the saccule in the vertical plane

A

e) Vertical forces are detected by the macula in the saccule in the vertical plane

Tilt of the head is detected by the macula in the utricle in the horizontal plane
Vertical forces are detected by the macula in the saccule in the vertical plane, e.g. elevation in a lift, and provide information on orientation of head when lying down.

301
Q

The superior and inferior colliculi are collectively known as the:

a) Corpora quadrigemina
b) Corpus striatum
c) Diaphragma sella
d) Mammillary bodies
e) Olives

A

a) Corpora quadrigemina
* Collectively, the superior and inferior colliculi are known as the corpora quadrigemina that sit on the posterior surface of the midbrain.

The caudate nucleus and putamen are collectively known as the corpus striatum*

302
Q

What EEG waveform has the lowest frequency but highest amplitude?

a) Theta
b) Beta
c) Sleep spindles
d) Alpha
e) Delta

A

e) Delta

MNEMONIC: B-A-T-S Drink Blood: You start with Beta waves, then Alpha waves, then Theta waves and then a non-REM Stage 2 you have Sleep Spindles.
From there, non non-REM Stage 3 is the D. So D for Drink. You have delta waves.
And finally, last is Blood, for beta waves in REM sleep.
(wakefulness -> REM sleep)

*Awake with eyes open - Beta (highest frequency, lowest amplitude)

Awake with eyes closed - Alpha

N1 - Theta

N2 - Sleep spindles & K complexes

N3 - Partial delta (lowest frequency and highest amplitude)

N4 - Full delta (lowest frequency and highest amplitude*

303
Q

What EEG waveform has the highest frequency but lowest amplitude?

a) Theta
b) Beta
c) Sleep spindles
d) Alpha
e) Delta

A

b) Beta

MNEMONIC: B-A-T-S Drink Blood: You start with Beta waves, then Alpha waves, then Theta waves and then a non-REM Stage 2 you have Sleep Spindles.
From there, non non-REM Stage 3 is the D. So D for Drink. You have delta waves.
And finally, last is Blood, for beta waves in REM sleep.
(wakefulness -> REM sleep)

*Awake with eyes open - Beta (highest frequency, lowest amplitude)

Awake with eyes closed - Alpha

N1 - Theta

N2 - Sleep spindles & K complexes

N3 - Partial delta (lowest frequency and highest amplitude)

N4 - Full delta (lowest frequency and highest amplitude*

304
Q

Where is Broca’s Area located?

A

Areas 44 + 45 (inferior frontal gyrus)

305
Q

Where is the Primary sensory cortex located?

A

Areas 3, 1, 2

parietal lobe -> postcentral gyrus

306
Q

Where is the Primary motor cortex located?

Where is the Supplementary motor cortex located?

A
  • Precentral gyrus*
  • Area 4
  • Area 6
307
Q

Where is Wernicke’s Area + auditory association area located?

A

Areas 41 and 42

superior temporal gyrus

308
Q

Where is the Primary visual cortex located?

A

Area 17

Occipital lobe

309
Q

Which of the following is FALSE regarding the supplementary motor area and premotor area?

a) The SMA and PMA are located in area 6
b) During thought ONLY blood flow increases in area 6
c) During thought and movement blood flow only increases in area 6
d) The premotor area innervates proximal motor units
e) The supplementary motor area innervates distal motor units

A

b) During thought ONLY blood flow increases in area 6
* During thought and movement, i.e. playing piano under PET imaging, blood flow increases in both area 6 (SMA & PMA) and area 4 (PMC)

Area 6 (SMA & PMA) ‒ planning the movement, increase in APs going to Area 4 (PMC)

Area 4 (PMC) ‒ activates neurones of CST and RST (motor neurons to muscles)*

310
Q

Which of the following is FALSE regarding long-term memory?

a) Declarative memory relies on the cerebellum
b) Procedural memory is associated with motor memory
c) Declarative memory is associated with semantic memory
d) Procedural memory relies on the cerebellum
e) Declarative memory relies on the hippocampus

A

d) Procedural memory relies on the cerebellum
* Declarative/Explicit Memory is a type of long-term memory that relies on the hippocampus - abstract memory for events (episodic memory) and for words, rules and language (semantic memory).

Procedural/Reflexive/Implicit Memory is a type of long-term memory that relies on the cerebellum and is independent of hippocampus - motor memory for acquired motor skills such as playing tennis, and rules based learning such as, in the UK, always driving on the left.*

311
Q

Which neurotransmitter is increased during REM sleep?

A

Acetylcholine

REM sleep is dependent on cholinergic pathways within the reticular formation and their projections to the thalamus, hypothalamus and cortex

312
Q

Which one of the following is a feature of A-alpha fibres

a) Largest myelinated
b) 30-70 m/sec velocity
c) Large myelinated
d) Transmission of slow pain
e) Unmyelinated

A

a) Largest myelinated

A-alpha fibres
Anatomy: Largest myelinated
Velocity: 70-120 m/sec
Function: proprioception, motoneurones

313
Q

What is the Gate control theory of pain?

A

Mechanoreceptors (A-beta fibres) and descending input (from Peri-aqueductal grey matter and Nucleus Raphe Magnus) activate inhibitory interneurones which act on A-delta and C fibres, preventing the sensation of pain

314
Q

Which of the following describes the changes in nociceptors in Allodynia?

a) Decreased threshold for response
b) Exaggerated response to normal and supranormal stimuli
c) Spontaneous activity in nerve fibres
d) No activity in nerve fibres
e) Increased threshold for response

A

a) Decreased threshold for response

315
Q

Which of the following describes the changes in nociceptors in Hyperalgesia?

a) Decreased threshold for response
b) Exaggerated response to normal and supranormal stimuli
c) Spontaneous activity in nerve fibres
d) No activity in nerve fibres
e) Increased threshold for response

A

b) Exaggerated response to normal and supranormal stimuli

316
Q

Where are 1st order, 2nd order and 3rd order neurones for C fibres?

A
  • 1st order: Dorsal root ganglion
  • 2nd order: Thalamus
  • 3rd order: postcentral gyrus (area 3, 1, 2)
317
Q

What are the Cristae in the Ear made up of?

Where is it located?

What is its function?

A
  • Cupula, hair cells and gelatinous material
  • Found in swellings at the beginning of each semicircular canal called the ampulla.
  • To detect rotational (angular) acceleration (same as the semicircular canals)
318
Q

What structural changes are associated with long-term memory formation?

A
  1. Increase in NT release sites on presynaptic membrane.
  2. Increase in number of NT vesicles stored and released.
  3. Increase in number of presynaptic terminals
319
Q

Which of the following is TRUE about the cyclo-oxygenase pathway and the sensation of pain?

a) Prostaglandins desensitise nociceptors to bradykinin
b) NSAIDs are analgesic because they stimulate cyclo-oxygenase
c) Cyclo-oxygenase converts prostaglandins to arachidonic acid
d) Cyclo-oxygenase converts arachidonic acid to prostaglandins
e) Prostaglandins have no effect on nociceptors
Correct answer

A

d) Cyclo-oxygenase converts arachidonic acid to prostaglandins

*Cyclo-oxygenase converts arachidonic acid to prostaglandins
Feedback
cyclo-oxygenase converts arachidonic acid to prostaglandins

In nociceptors, local chemical mediators e.g. bradykinin, histamine,
prostaglandin can cause depolarization and generation of action potential that carries sensation of pain

Prostaglandins are molecules produced by the cyclo-oxygenase pathway that sensitise nociceptors to bradykinin

Non-steroidal anti-inflammatory drugs (NSAIDs) are analgesic (and antipyretic & anti-inflammatory) because they inhibit cyclo-oxygenase which converts arachidonic acid to prostaglandins*

320
Q

Which neurotransmitter controls the descending system of peri-aqueductal grey and rosteroventral medulla?

A

Noradrenaline

321
Q

Which neurotransmitter controls Sleep induction and Initial stages of sleep?

A

Serotonin

322
Q

How long does REM sleep last for?

a) 90 mins
b) 5-30 mins
c) 30-60 mins
d) 10-20 mins
e) 120 mins

A

b) 5-30 mins

* On average, REM sleep occurs every 90 minutes, with increasing duration through the night and lasts for 5-30 mins*

323
Q

Which long reflex involves rapid postural adjustments that are made to stop you falling when you trip?

A

Dynamic righting reflex

reflexes that involve rapid postural adjustments that are made to stop you falling when you trip.
→ Long reflexes, involving extension of all limbs. Most profound in cats

324
Q

Which type of pain is described by the following: ‘‘pain that arises from altered nociception despite no clear evidence of actual or threatened tissue damage’’

A

Nociplastic pain

325
Q

What are the constituents of the papez circuit?

A
  • Hippocampus
  • Mammillary bodies
  • Anterior thalamus
  • Cingulate gyrus

(Responsible for Episodic memory)

326
Q

What are the Otolith organs?

A
  • Utricle

- Saccule

327
Q
Which one of the following is a feature of A-beta fibres
0/1
a) Largest myelinated
b) 30-70 m/sec velocity
c) Transmission of fast pain
d) Unmyelinated
e) Transmission of slow pain
A

b) 30-70 m/sec velocity
* A-beta fibres

Anatomy: Large myelinated

Velocity: 30-70 m/sec

Function: touch, pressure, vibration*

328
Q

What is included in the….

  • Eye
  • Voice
  • Movement

… section of the GCS?

A
  • Eye:
    1. No response
    2. To pain
    3. To speech
    4. Spontaneous
  • Voice:
    1. No response
    2. incomprehensible sounds (moans, groans)
    3. inappropriate words
    4. confused (dk where they are)
    5. Orientated to time, person place
  • Movement:
    1. No response
    2. Abnormal extension
    3. Abnormal flexion
    4. Flexes to withdraw from pain
    5. Moves to localised pain
    6. Follows commands
329
Q

What is the function of the Utricle?

A
  • Detects back/front tilt and horizontal acceleration.

ie. in a car

330
Q

What is the function of the Saccule?

A
  • Detects vertical acceleration

ie. in a lift

331
Q

What are the effects of the ANS on the heart?

A

Remember b1 receptors are only found in the heart!!

Sympathetic effect on the heart:
- Noradrenaline activates b1 receptors on the pacemaker cells
→ Increases heart rate
- Noadrenaline activates b1 receptors on the myocytes
→ Increases strength of contraction

Parasympathetic effect on the heart:
- Acetylcholine activates muscarinic receptors on pacemaker cells
→ Decreases heart rate
- Acetylcholine has a small effect on myocytes
→ Little to no effect on strength of contraction